You are on page 1of 76

Contract Law Most Important Principles

Freedom of Contract Sanctity of Contract (that it is binding)


1. 2. 3. 4. Issue-Who/What/Why Principle/Law (any exceptions) Application of it (all possibilities) Conclusion

Agreement
The test for whether or not an agreement has been made is objective rather than subjective to encourage certainty in commercial transactions and to avoid difficulties in trying to inquire into the state of mind of the party to a contract. Doctrine of Mistake-If the offeree wrongly assumes he is getting something that the offeror did not promise, he cannot end the contract when he doesnt get this.. It is all down to what the reasonable man would expect from a contract. Smith v Hughes-selling of oats. The law does not allow a party to snatch at a bargain which he knew was not intended. Harthog-mistake in offer price for piece/pound. Role of Fault o Denny v Hancock o Wanted to buy property with misleading place for it. o Refused to buy when he found out the mistake which was upheld by the courts.

WHEN LOOKING AT WHETHER A CONTRACT HAS BEEN MADE OR NOT YOU HAVE TO LOOK FOR: 1. 2. 3. 4. Agreement Consideration Certainty Intention

Unilateral/Bilateral Contract Bilateral-A contract with promises on both sides. Unilateral-One where only one party makes a promise and requests an act form the other party in return. Here the performance of the contract is both acceptance and consideration.

Offer

Whether something is an offer or not depends on the intention of the offeror as it appears from his behaviour to a reasonable offeree. The word offer is not necessary for an offer to actually arise in law, even if contained in an advertisement. Invitation to treat-goods on display in a shop, adverts for goods and even menus are not generally treated as offers-this is because the demand for the goods might end up exceeding the supply. Partridge A shop can only refuse to sell to individuals not groups. Offers can be withdrawn at any time before it is accepted. Whether it is an offer or not depends on the intention of the seller as it would appear to a reasonable buyer. If an offer says subject to contract, there will be no contract until this restriction is removed. Gibson v Manchester City Council-This case which dealt with the sale of council houses held that there was no contract as although one had decided to sell and the other decided to buy, due to the language used by both (the word may was used), there was no formal agreement (meeting of the minds) Once the performance has begun, the offer cannot be revoked but instead allows the offeree to complete the performance. Offers for rewards-notifications for acceptance is waved. Consideration-does not always have to be money or goods-can also be the following of instructions e.g. offering someone money to walk backwards to York

Tenders A tender is an invitation to negotiate and not an offer-Spencer v Harding An example is someone who invites many builders to come and build something for them. They are not obliged to accept the lowest or highest offer. Depending on the case they are obliged to at least consider all of the tenders. Sometimes damages can be awarded for a loss of chance when it comes to tenders. Blackpool and Fylde Aero Club Ltd v Blackpool Borough Council-tender to operate pleasure flights. Tenders had to arrive no later than noon on March 17th. By mistake D thought Cs tender was late and discounted it. C sued and won for loss of chance. An invitation to tender is not generally an offer but the tenders themselves are. (unless you state the goods will be sold to the highest tender etc) This is because if their was only one tender, the seller would be bound to sell which is quite unreasonable. Referential Bids-only allowed if the invitation clearly allows referential bids as opposed to fixed bids. Problem is that two people may make exact same bid-no way of deciding which is the highest bid-need a certain mechanism. Harvela Investments v Royal Trust Co of Canada Auctions

The highest bid on an auction cannot be rejected simply because it is considered not high enough. In Barry v Davies we see this when the auctioneer refused bids at auction because they were too low and ended up selling to another party. A sale by auction is complete when the auctioneer announces its completion-this is the agreement to the bidders offer.

Duration of Offers If no time limit is specified, there is still a reasonable time limit in play. This varies according to the subject-in a volatile situation (stock market) there would be much shorter time limit than in the sale of a car. Even if someone specifies a time limit within which acceptances can be made, the offer can still be revoked the next day unless someone has already accepted-Routledge v Grant. Byrne v Van Tienhoven (1880) 5 CPD 344-In Byrne v Van Tienhoven, there was a contract even though the two parties were in disagreement. This is because the offeror had made an offer but then changed their mind and sent a revocation. However, the other party had received the offer before the revocation reached them and they sent their acceptance.

Acceptance
Contracts are legally binding as soon as the offer has been accepted regardless of whether the actions of the contract have been performed. If an acceptance attempts to vary the terms of an offer, it is seen as a rejection of that offer and a counter-offer. Acceptance also requires knowledge of the offer beforehand. (German Law is the opposite of the previous two points).

Communications of Acceptance (Non-postal) Acceptance is only affected when the offeree communicates with the offeror. With instantaneous communication-the acceptance is regarded as affected when it is heard. Sometimes its uncertain whether the offeror has actually received the acceptance e.g. whether the fax was actually read or not. When fax or telex is used it is treated as received when it has arrived in the building. The Brimnes Silence-you cannot force a contract on someone and silence cannot be a method of acceptance unless the offeree has accepted by conduct that isnt communicated to the offerror (such as misdirected post) or unless the offeree cheerfully accepts the silence. o Felthouse v Bindly-transaction between uncle and nephew for a horse. The uncle who was the buyer tried to use silence in his offer.

Eaglehill Ltd v Needham Builders. Here it was held that a notice is received when it is opened in the ordinary course of business or when it would be opened if the ordinary course of business was followed Which Jurisidiction?-Entores v Miles Far East Corporation-Here the claimants made an offer by telex to defendants in Amsterdam who accepted by telex. However, disagreements came about as to where the contract was made and therefore which jurisdiction was in play. It was held that the contract was formed when the acceptance was received in England and therefore English Courts had jurisdiction.

Battle of Forms Sellers and buyers both want to do contract in their own terms. A seller might include an invoice with goods containing a price escalation clause (their terms). The buyer might stamp it stating that the goods were received on their terms. Generally, contracts favour the seller. A reply to an offer with additions or modifications is a rejection of the offer and constitutes a counter offer. A basic rule is that the offeror is master of his offer and can specify any method of acceptance. Butler Machine Tool Co. Ltd v Ex-Cell-O Corporation Ltd o Claimants had quoted a price of a machine tool with delivery to be given in 10 months. Included was a price variation clause which provided for rise in price is there was a rise in costs. o The buyers rejected this and relied on their own terms and conditions which they had set out which ended up being different to those contained in the offer. o Tool not delivered for 16 months and costs had increased that the sellers claimed an extra 2892. o Held that the buyers had rejected the original offer and made a counter offer which the sellers subsequently accepted by signing an acknowledgement. Contracts by Post Once the letter is posted the contract comes into effect (only applies to acceptances). This is unless the offeror states specifically that the acceptance is not counted until the letter is received. However revocation is not counted until receipt. This was affirmed in Household Fire Insurance Co Ltd v Grant. This rule also does not apply if the letter is misaddressed (Korbetis) or if there is a postal strike on at the time that the letter was sent (Holwell v Hughes) If there is a daft situation the postal rule does not apply. The reception rule is seen to hinder the offeree in his expectation that a contract has been formed but that the postal rule can lead to the offeror getting into a contract that he didnt know about.

E-mail-It is generally accepted that emails are an instant form of communication and so the postal rule does not apply-the contract is made when and where the email arrives.

Unilateral Contracts Most unilateral contracts are made up of public offers such as large competitions (cannot tell who read the advert and cannot contact them)you withdraw the offer by giving similar publicity that you did to the offer. This was held in the obiter of New Zealand Shipping Co v A.M Satterthwaite & Co. at a reasonable period before the time for performance. Finality of Agreement Agreement to agree or agreement to negotiate is not legally binding. British Steel Corporation. Letters of intent are not specific enough to be legally binding. Contract for sale of goods and contract fails to fix the price, the court will impose a reasonable p rice.

Basis of Enforcement of Contracts


1. 2. 3. 4. Intentions of the parties Use of special formalities Element of bargain No vitiating elements (duress, misrepresentation, legality of purpose etc)
No contract is made if there is no intention for it to be legally binding e.g a domestic case or a commercial case which is pre-decided not to be binding. Domestic Cases o Balfour v Balfour-Husband works abroad, promises to pay wife 30 a month, ends up breaking promise. Was decided that the agreement was not binding as the parties didnt intend them to be. o Reasons-This is to prevent opening the floodgates and also due to the that the law should be regulate family relationships. o However, domestic agreements can be binding contracts if they are intended to be or if there are significant consequences from it. o Merritt v Merritt-Here it was legally binding as the couple were about to separate and so they were bargaining keenly with the intent for it to be binding. Commercial Cases-Esso v Commissioners of Customs-Esso give petrol stations world cup coins with one to be given out after ever gallon of petrol. However, the commissioners tried to charge tax on it. It was seen as binding as Esso did envisage a bargain. o Mere puff-Carbolic Smoke Ball-It was seen that the defendants intended the contract to be legally binding due to the deposit of the reward money. o Honour clauses-just because the parties proclaim their intention does not mean that it is legally binding. Rose and Frank v Crompton and Bros. o Letter of comfort-provided by a company to a financier to induce them to provide funds.

Intention

Form
Generally contracts do not need to be in any particular form but there are exceptions: Reasons for formal requirements o Evidential reasons o Cautionary reasons-this may lead to greater care when entering into a contract.

1.

2. 3. 4.

o Protective functions-may help to protect the weaker party in certain types of contract e.g. contract of guarantee in writing. Promise by deed (binding even though there is no consideration)Law of Property (Miscellaneous Provisions) Act 1989 a. Document called a deed or indicating it is intended to be a deed. b. Signed by maker. c. Attested by witness. d. Conduct showing intention for it to be bound. Contract for sale or disposition in land-must be in writing. Contracts of Guarantee-must be in writing Consumer Credit Agreements-must be carried out in the way that is laid down in the statute. Why have formal requirements-Helps to promote certainity, encourages care to be taken before contracts and helps protect weaker parties. Atiyah-the insistence on form shows our weak legal system as there is a lack of confidence in ability of courts to find out truth without trappings of a form. They are also seen as time-consuming and clumsy.

Nature of Consideration

Consideration

This is a detriment to the promisee and/or a benefit to the promisor. It is an exchange of promises with a bilateral contract. It is the exchange of a promise and performance in a unilateral contract. It helps to prove intention-acts as a badge of enforceablilty. It also helps to stop judges personal opinions coming into the picture. Atiyah-Consideration is there to ensure that there is a good reason for enforcing a promise. Treitel-Disagrees with Atiyah, thinks that his view leads to uncertainty. He says that the Doctrine of Consideration is used to help predict future decisions. The requirement is that consideration must move from the promisee. A promises 100 to B in return for B sending goods to C. Here B (promisee) suffers a detriment but there is no benefit to promisor (A). Consideration must be in the minds of the parties at the time the contract was drawn up. Combe v Combe-After divorce, allowance promised at 100 and wife didnt apply for maintenance. The husband then refused to pay. There was clear reliance on the agreement, however, it was not enforceable because there was no exchange of consideration. Currie v Misa-It was found here that consideration may consist either in some right, interest, profit or benefit accruing to the one party or some forbearance, detriment, loss or responsibility given suffered or undertaken by the other. If there is a contract variation, there must be fresh provision of consideration. Doctrine of Privity-only the parties to a contract can sue on it (common law). May lead to the exact same result as Doctrine of Consideration. Tweddle v Atkinson-Husband and wife married. Both fathers agree contract together and agree to pay husband and say he can sue if they do not pay. Both fathers end up dying and the father-in-law hadnt paid and the husband sued. His action failed because no consideration moved from the husband and also that he wasnt a party to the contract. However, after the Contracts Act 1999, third parties are given statutory right to sue on a contract between others. However, the two parties to the contract must still have provided consideration. If someone, furnishes consideration on behalf of another, it is counted a good consideration.

Sufficiency of Consideration

For consideration to count it must be something of value in the eyes of the law. Thomas v Thomas-Plaintiffs husband expressed wish that if she survived him, she should have use of their house. After death, executor agreed to allow this-due to husbands wishes and also on agreement to payment of 1 a year by thr wife. Only the latter was counted as consideration. The courts do not care about the adequacy of the consideration as long as it is something of value. A gratuitous promise may become binding if there is some nominal consideration. Intangible Returns o Hamer v Sidway-Consideration provided by promise by nephew to give up drinking, smoking, swearing and gambling in return for money from his uncle. o Even a promise to give up a nuisance can still count as good consideration. o There is no consideration if the promise made was never intended to be carried out. D & C Builders v Rees-Owner forced builders who badly needed money to accept lesser amount by offering them that or nothing but he was not allowed to do this and was forced to pay the full amount. Held that when a debtor pays a lesser amount than the amount owed, that it does not count as settlement of the debt. Ward v Byham-Husband and wife divorce. Husband agrees to pay the mother 1 a week provided the child was well looked after and happy. After a few months he stopped paying after the mother married. Denning held that their was good consideration by the mother for the fathers promise Generally, if something is imposed by law it can not count as good consideration.

Past Consideration (Generally does not count) Even if one party gives consideration to the other due to past consideration they received, it does not count as a binding contract as there is no reciprocity. However, if the past act was done at the promisors request, he parties acknowledged that the act would be remunerated by payment of benefit and the payment had been enforceable then the consideration may count. Eastwood v Kenyon-The plaintiff was executor of account and had to look after the dead mans daughter. He then borrowed money to accomdte this. When the daughter got married, her husband promised he would recompense Eastwood. However, it wasnt binding as the consideration for it (bringing up daughter) was furnished before the agreement. Forbearance to sue and compromise of suit Contracts can easily be made through forbearance to sue-Cook v Wright. In debt cases, if A promises security and B agrees not to sue in the case of the repayments not being met, there is a binding contract.

However, there must be a forbearance in fact if it is to count towards consideration. If there is a claim that will fail but is believed valid, the promise to forbear suing on it can still be good consideration. The decision is completely subjective and rests on belief of whether or not the claim will succeed. If a claim is known to be bad then forbearing to sue can still act towards consideration as it has a nuisance value but the deceitful party will still fail due to wrongful duress (public policy).

Performance of an existing duty If a person does or promises to do something that he was already bound to do. We must distinguish a duty owed to the public, a third party and to the promisor. Until Williams v Roffey, the performance of an existing duty to the public or the promisor was not consideration. Duty to public o If the police agree to protect someone for money, this would not be allowed to count as consideration as they already have a duty to the public. o As all citizens have a duty to attend court if required as a witness, someone who offers to pay his witness to make up for lost working time would not count as consideration. o However, if someone with a duty to the public goes much further than they were reasonably expected to do, it may very well count as consideration e.g whether Sheffield united had to pay for extra police protection during football matches. The answer is yes if they would be providing a special service far above the duties normally expected of them. o According to Lord Denning, the performance of an existing duty to the public can be consideration as long as it does not negatively affect the public. Duty to third party o Scotson v Pegg-Contract between A and B that A should deliver coal to C. C agreed that within consideration of the deliverance of the coal that he would unload. This is was action of A on Cs promise. This was counted as consideration even though he had a contract with B. Duty to the promisor o If you can show an old contract has been abandoned and a new one put into place than you can show consideration. o Stilk v Myrick-Seamen were to be paid for a voyage from London and back. At destination, two of the crew deserted and the captain split their wages amongst the others and later refused to pay. Held that it was not a binding contract as there was no consideration to support it. o Hartley v Ponsonby-Ship left England with 36 crew. Arrived 6 months later and many deserted. Captain promised extra money for the crew to continue going. The seamen were not bound to re-

embark as the ship was so short-handed. So the original contract was discharged and the new agreement was supported by consideration. Practical Benefit-Williams v Roffey (Promisor to pay more) o Williams entered into sub-contract with Roffey to carry out carpentry works in flats they already had a contract to build. Williams then got into financial difficulty and had underpriced the job at 20,000. The main contract had a time penalty clause. Roffey (made suggestion) was worried they would suffer if the work was not complete and so made an oral agreement to pay Williams 10,300 more. After a few weeks and after 8 more flats had been substantially completed, the defendants had made only one further payment and so Williams ceased work and sued for the additional sum. At first instance, held that there was consideration for promise to pay more. In Court of Appeal, this was also held. It was seen that Roffey received many practical benefits including not having to find a new carpenter, not incurring the time penalty etc. Also the absence of consideration was not even submitted. o If there is a contract, doubt that A will carry out their duties and B promises extra money to A to carry out duty with B obtaining a benefit without duress or fraud then it is capable of being consideration. o We can almost say that we need consideration for the making of a contract but not for its variation. o View taken abroad New Zealand-Williams v Roffey took into account practical benefit as opposed to legal niceties. Part payment of a debt Pinnels Case o Pinnel sued for a debt due. The defence was that he had paid Pinnel a lesser sum in full satisfaction of the debt. o As the sum was paid early, it was found that there was consideration as there was a practical benefit to Pinnel. o If a lesser sum was paid on the day, it would not count as consideration. o However, there must be full acceptance of this by the other party. Foakes v Beer. (Promise to accept less). Part payment of a debt is not good consideration for a promise to discharge the entire debt. In this specific case the claimant wanted to claim the interest owing to her. This was found to be without consideration, and does not prevent the creditor after payment of the whole debt and costs from proceeding to enforce payment of the interest upon the judgment. Re Selectmove Ltd o Inland revenue agreed to accept tax by instalments by a company that owed it a lot of tax but the court said there was no consideration to accept this and so they were not bound to do so.

o It was suggested there was a practical benefit to Inland revenue using Williams V Roffey as an example but the court held it was only applied in the sale of goods/services (not in the payment of a debt) as it would run counter to Foakes v Beer otherwise.

A contract which is obtained by improper pressure is voidable. The pressure can be by way of actual threat of violence but economic pressure can also suffice. There must be: 1. Lack of consent/coercion of will 2. Illegitimate pressure
In Universe Tankships Inc of Monrovia v International Transport Workers Federation [1983] 1 AC 366, the House of Lords held that the victims apparent consent must be coerced by pressure which the law does not regard as legitimate. Here the tugmen (through their trade union) refused to work until they received more money which was a breach of their contract.
Illegitimate Pressure: 1. Threat to do something unlawful-threat to break contract in order to extract more money-The Atlantic Baron [1979]-wanted extra money for building a tanker due to drop in value of dollar. The other party agreed as they need the tanker. Although the pressure was illegitimate they lost as they made the payments without protest and took too long to bring the claim. 2. If the threat itself is unlawful Lawful threat made in good faith-CTN Cash and Carry v Gallagher Ltd [1994] 4 All ER 714-Defendant supplied Cash and Carry with cigarettes and allowed them credit. On one occasion they delivered to wrong warehouse and were subsequently stolen. Defendants still insisted on full payment and threatened to withdraw credit facilities. They genuinely thought they were entitled to the payment (even though they were not in fact). Claimants gave in and paid but then sought repayment under duress. Held by the court of appeal that the defendants could withdraw credit for any reason and so the claim failed. If there is simply negotiation taking place and you strike a hard bargain which the other party gives in, duress will not succeed-Williams v Roffey Borrelli v Ting-Company in liquidation. Liquidators agreed to wave all claims against company as long as they did not let the scheme of

Economic Duress

arrangement through. It was held that the contract was voidable for duress.

Coercion Pao On v Lau Yiu Long [1980] AC 614-Lord Scarman-necessary to enquire whether the coerced person protested, had an alternative course open to him such as an adequate legal remedy, if he was independently advised and if they took steps to avoid the agreement. A-G for England & Wales v R [2002] 2 NZLR 91 (CA) o R was soldier in UK and member of the SAS. He signed a confidentiality contract with Ministry of Defence not to reveal information he acquired. o After leaving SAS he wrote a book about his experiences and the Attorney General tried to get an injunction to its publishing. o The contract was found to not contain illegitimate pressure and that it wasnt coercive as he wasnt forced to sign it, it was just unpalatable.

Promissory Estoppel
Prevents the makers of a contract going back on it even though there is no consideration.
Central London Property Trust Ltd v High Trees House [1947] KB 130 Under, Central London Property Trust v High Trees House, a promise intended to create legal relations which is intended to be acted on and is acted on is binding in law, although it is difficult to find any consideration. o In this case we find the plaintiff company agreeing to let a block of flats to the defendants. o Due to the war, many of the flats were not leased and it was clear the defendants could not keep up rent. o The plaintiffs agreed to reduce the rent (they made the suggestion). The defendants continued to pay reduced rent for the coming years even though all of the flats became occupied. The plaintiffs then explained that they needed to pay the full rent owing to them. Denning held that they had intended to create legal relations between themselves and the promisee acted upon it and so therefore it was binding even though there was no consideration present. However, it was only intended to be binding for under the specific circumstances at the time when the flats were not being let and so the court held that the defendants owed the plaintiffs. High Trees Principle o When by words or conduct, a person make an unambiguous representation as to his future conduct and it is relied upon, he will not be able to act inconsistently with it if by doing so it creates prejudice. Hughes v Metropolitan Railway-Lord Caine-If parties enter negotiation which leads one of the parties to suppose that rights under a contract will not be enforced, the person who wishes to enforce them will not be allowed where it would be inequitable. o Here the defendant lessees were asked by a notice from the owners to repair their property and gave them time limit. o They then offered the interest in the property to the other party and advocated that the works be put on hold until they had reached an agreement. o The other party began negotiations but did not respond to the issue of deferring the repairs.

o When the talks failed the other party attempted to kick out the defendants as the repairs had not been finished in the time limit. o They were estopped from doing so as they had led the lessees to believe that the time limit for the repairs was suspended. Key Elements for estoppel 1. Clear and unequivocal representation (High Trees) 2. Reliance by promisee-Sometimes the promisee must have acted to his detriment. Hughes-lost time for the repair. 3. Inequitable to go back on promise-often shown by promisee acting in reliance, not fair. D & C Builders v Rees-Owner forced builders who badly needed money to accept lesser amount by offering them that or threatening to break the contract but he was not allowed to do this and was forced to pay the full amount. Held that when a debtor pays a lesser amount than the amount owed, that it does not count as settlement of the debt. Collier v Wright-3 partners who owed money to Wright. Wright suggested that Collier could pay his 3rd and his obligations would be distinguished. 2 of the partners went bankrupt and Wright came to Collier looking for the money. The Court of Appeal held in a preliminary hearing that although the payment of a lesser sum was not legally binding as there was no consideration, they invoked the doctrine of estoppel and said that if Collier relied upon this promise it would be inequitable to go back on it. 4. Effect generally suspensory-less effect than that of a binding contract. However, sometimes in cases such as High Trees it may have a permanent effect if it is particularly inequitable. Hughes-still had 6 months to carry out the repairs. 5. A shield, not a sword-cannot be used in a positive was to sue (unless estoppel by convention or proprietary estoppel) but only as a defence. a. Combe and Combe-Husband promised to pay maintenance. No consideration. Held at first instance, as the wife had relied upon it she could sue on it. In the Court of Appeal they held that this wasnt the case as this would create a new cause of action. Estoppel by Convention/Representation Depends on a common assumption operating between the parties, not upon a representation made by one to the other. This is concerned with the existence of a promise. Amalgamated Investment and Property Co v Texas Commerce International Bank o Were certain loans covered by a guarantee. o The court held that the parties entered into guarantee under the shared assumption that it did cover such loans. o The estoppel here was prevent the claimants denying the guarantee. o Here the defendants used estoppel as a shield.

Proprietary estoppel If an owner of land leads another to believe that they have an interest in land and in faith of this the other spends money on the land, the owner may be prevented from denying that interest exists. Dillwyn v Llewelyn-Father promised son land, no consideration. Son spent money and built house on land. The son sued on estoppel to have the land conveyed to him Overseas In Australia, promissory estoppel can give rise to action. Walton Stores v Maher-Maher would demolish an old building and build a new one specifically for Waltons. No contract drawn but negotiations begun and draft done up. Maher started building and then Waltons tried to back out. High Court held that Waltons led Maher to believe that a contract would be signed and as they he had relied on this, it would not be fair to go back on it. Maher ended up getting damages.

That a contract cannot confer rights or impose obligations arising under it on any person except the parties to it. Only the parties can sue or be sued. However, this common law has been substantially modified by legislation. (Contract Rights of Third Party 1999 )
Introduction Dunlop Pneumatic Tyre v Selfridge. o Dunlop sold tyres to Due as long as they wouldnt sell them below a minimum retail price. Due agreed that when selling to trade customers, it would sell on under the same terms. o Due sold on to Selfridge who promised to agree to this restriction and promised 5 to Due for each tyre sold in breach of this agreement. o Selfridge then sold below minimum price. Dunlop took an action for the 5 per tyre. o Held that it was irrecoverable. Only someone who is a party to a contract can sue on it. Remedies of promisee 1. Specific Performance (Discretionary) a. Here the third party can gain a benefit. b. Beswick v Beswick-Coal merchant transferred business to nephew who promised to pay 5 a week to the aunt after this death. The nephew did not pay once uncle died. Widow sued in her own right and also on behalf of her husbands estate. The claim in her private capacity failed as she was not a party. However, she succeeded in her action on behalf of the estate and got an order for specific performance. However, her case relied on whether or not the estate were willing to sue. 2. Declaration (Discretionary) a. Snelling v John Snelling Ltd-3 brothers who own family company with the company funded by loans by all three. The brothers agreed between themselves that they wouldnt demand repayment at a certain time. They also agreed that if any resigns, they would forfeit the loan. One of the brothers did resign and sued the company for repayment. The company wasnt party to the agreement and so was liable to repay. However, the other brothers could enforce the promise. Court gave a declaration that it was binding. 3. Damages

Privity of Contract

The courts do not want the promisor to be free from liability-would create a legal blackhole. Damages for 3rd partys loss General Rule-cannot claim damages for loss of someone else. Woodar v Wimpy ExceptionsJackson v Horizon Holiday Ltd o Plaintiff organised holiday in Sri Lanka with defendants company. o He was promised high standard accommodation with a golf course, salon, pool etc but none of this materialised. o Holiday was disaster and sued for breach of contract. o Held that the plaintiff could recover, not only for his own loss, but that suffered by his wife and children. Although he was not a trustee to them and he was the only party to the contract, he had entered into the contract partly for their benefit. o The House of Lords supported Dennings choice of outcome but disputed his reasoning. Dunlop v Lambert o The consignor of goods lost at sea could receive damages from carrier even though ownership at the time had passed to the consignee. The Albazero o In a commercial contract concerning goods, the original party can sue for the loss suffered by the consignee if the goods are lost by the carrier at sea. o However, in this case there was already a 3rd party right of action through a bill of lading and so they could not sue. Linden Gardens Trust v Lenesta Sludge Disposals o Here the lessees entered into a contract with the contractors to remove asbestos from their building. o They then assigned their rights of action in regard to the property to the plaintiff without the contractors consent. However, they are not allowed to do this. o The contractors failed to remove the asbestos and the plaintiffs wished to sue for breach of contract and loss suffered by the lessees. o Same rule as in The Albazero but applied to property. o Held here that the plaintiffs could recover damages for the loss of someone else. It was clear that he would pass it directly to the 3rd party. This is because a party to a contract might have a genuine commercial interest in ensuring that contractual relations with the party he selected were preserved. Darlington BC v Wiltshier Northern

Used Linden Gardens. A entered into a building contract with B to build a recreational centre for the benefit of C. A then assigned all rights and causes of action to C. C then wished to sue B for breach or contract. Held on appeal that C could claim as both parties where aware that the building contract was entered into for the benefit of C and that it was foreseeable that breach of contract would cause damage to C. Alfred McAlpine Construction v Panatown o McAlpine made contract to build office block for Panatown but the site belonged to a third party (Unex). o McAlpine entered a duty of care deed with Unex, accepting direct liability for negligence. o There was a defect in the building and Panatown sued McAlpine. McAlpine argued that Panatown had suffered no financial loss. o Held that the claim failed because the Albazero exception didnt apply when the 3rd party itself (Unex) had contractual rights and direct right to claim. The majority did not want to unjustly enrich the promisee. o Dissenting judgments said that Panatown should get compensation for loss of expectation.

No right of action for a third party in tort. Only contract allows you to have a remedy in regards to the quality of property. Damages for the promisees own loss The core problem is whether your loss is nominal or substantial. Price v Easton o A promisor who failed in breach of contract to pay debts owed by the promisee to the third party. o Held that the promisee could claim for substantial loss. In a cases such as Beswick (above) it was held that if the contract was not specifically enforceable only nominal damages would have been awarded as the uncle had no other assets and so no alternative provision could be made. If there had been, the estate might have made a voluntary contribution towards the widow-a loss which could be recovered from the promisor. You might even be compensated simply because you wished the third party to receive a benefit from the promisor but had to ensure that they did so by alternative means. Worry that a promisee will sue for 3rd partys loss and then pocket the money themselves. To combat this the court could ask for a formal undertaking to the court to pay the money to the 3rd party.

Trusts of Contractual Rights This is an equitable obligation to hold property for someone else. The 3rd party can enforce their benefit.

You can also be a trustee of a chose in action-a thing in action. An example is having the benefit of a debt. You have a right of action and can sell it on even though it is not a physical thing. The same applies to a contract If you declare yourself a trustee to someone else with a benefit to a third party, the third party can sue. For this to happen, there needs to be good evidence that this trust was intended to be created.

Exclusion clauses and third parties Where there is an exclusion cause giving a benefit to a third party, can they enforce that benefit? If you make a contract and exclude liability for yourself and a third party, can the third party rely upon this? Only in the following ways: Principal of Agency-Person requires rights where an agent makes a contract on your behalf. o Scruttons v Midland Silicones A drum of chemicals shipped to London which included a clause that limited the liability of carrier. Stevedores employed by carrier damaged the goods and the Stevedores then claimed limited liability from the exclusion clause. Held that they were not parties to the contract and so could not gain the benefit of the exclusion clause but that in different circumstances they could i.e. if they had made a contract as in NZ Shipping Co v AM Satterthwaite (The Eurymedon)-in this case there was a bill of lading which protected the carries as well as the stevedores. Defences to tort actions 1. Vicarious immunity-Agents acting under a contract with an exclusion clause can not be sued but can get the benefit of their employers contract. Elder Dempster & Co v Paterson Zochonis- a bill of lading which protected the people who worked for the company. (before Scruttons). However, it was held as unsound in Scruttons, yet it was approved again by Supreme Court of Canada in London Drugs Ltd v Kuehne & Nagel (employees damage goods when there was a $40 limitation clause in place with their employers and the other party-held that the employees could get benefit of the contract). 2. Assumption of risk-by entering into a contract excluding the liability of a 3rd person, then the contracting parties assume the risk of the 3rd partys negligence. For this to be present there must be a full knowledge of the risk. 3. Non-contractual notice-a person is free to exclude a tort liability by giving this notice. Smith v Bush-house purchasers applied for mortgage from building society who instructed valuers to carry out an evaluation. The application had a disclaimer liability which covered the building society and the valuer. Held that the exclusion notice was sufficient

under the common law for the valuer to rely on but not under the Unfair Contract Terms Act 1977 4. Exclusion of liability as a benefit under the Contracts (Rights of Third Parties Act 1999). Principle of Novation Takes place when two contracting parties agree that a 3rd person shall stand in relation of either of them to the other (replace one of the contracting parties). This creates a new contract but all 3 partied must agree. Third party claims in negligence White v Jones-Solicitor did not execute a will in good time and so the legatees suffered as a result. Under Murphy there is no tort action for defective buildings bought by a third party. The Contracts (Rights of Third Parties) Act 1999 A Burrows-The Contracts (Right of Third Parties) Act 1999 and its implications. Provides for a significant exception to privity. A third party may enforce a term in the contract if the contract expressly provides that he may or of the term indicated that it will confer a benefit on him (unless it appears that the parties did not intend the term to be enforceable by the third party). The 3rd party must be expressly identified by name, as member of a class or answering a particular description but need not be in existence e.g. unborn child. This is good as it allows the courts to look at implied conferral of rights as opposed to just to express rights. o This is good as not all contracts are well drafted. o It also helps people such as those in Jackson. But it makes sure not to step on the autonomy of contracting parties too much as they can make express decision in the contract that the 3rd party is to have no contractual rights. The act also eliminates the need for 3rd party consideration. However, there can be unintended liability as parties cannot now leave a contract in a neutral stage as a 3rd party right of action may be found even if it was not intended. Also, it makes it difficult for the contract parties to vary the contract without the 3rd party assent which almost puts the 3rd party in a stronger position than the original parties.

Undue Influence
A relationship that involves one person with a dominating influence over another and uses it to advantage himself. It can depend on exploitation of a vulnerable person and/or someone whose consent has been impaired. However, the doctrine is not there to save people from their own folly.
Nature of Undue Influence
Categorisation-relationship of trust and confidence-Barclays Bank v OBrien per Lord Browne-Wilkinson o Should there be a categorisation of undue influence? o Cases should be divided into Class 1 (undue influence shown to have happened-must be proven affirmatively) and Class 2 (presumed to have happened-must prove special relationship of trust of confidence and the transaction was manifestly disadvantageous to the claimant.) o Class 2 (A)-presumed as a matter of law (solicitor and client/doctor and patient/trustee and beneficiary/parent and child etc) o Class 2 (B)-must be proven on facts of the case. Manifest Disadvantage-National Westminster Bank v Morgan o Confirmed that the impugned transaction should be wrongful in that it constituted a manifest and unfair disadvantage to the person seeking to avoid it. o Held that there was no dominating relationship between woman and a bank manager and that it hadnt been disadvantageous to the wife when she gave the charge over the house to prevent repossession of her house. o Later extended to only cases of presumed undue influence. N.B Present Law-Royal Bank of Scotland Plc v Etridge (No.2) (JUDGMENT OF LORD NICHOLLS) o Equity had identified two forms of unacceptable conduct: 1. Acts of improper coercion 2. Relationship where one has acquired ascendancy over another and takes advantage of that. o Proof needed: 1. Complainant placed trust and confidence in the other party.

2. A transaction which calls for an explanation. o Disadvantage is relevant but not an essential factor. o The legal burden remains on the claimant to prove undue influence as the courts dont want people to just be able to get out of contracts. However, the evidential burden may shift. o Certain cases where you can presume influence (class 2 A)- legal relationships like solicitor/client but not husband/wife but cannot presume that it has been abused. Want to keep the list as open as possible. Class 2 B-husband/wife or employer/employee requires a manifest disadvantage. o Sometimes undue influence is proven directly by evidence or exertion of wrongful influence but sometimes circumstances are just proven which justifies the drawing of an evidential inference. Proof of Undue Influence Actual undue influence o Snee-A son got his alcoholic/mentally incapable mother to grant a mortgage to secure his loan. Allcard v Skinner o Woman gave major property gifts to her sisterhood. o When she joined, the rules provided that all property should be given to relatives, the poor or the sisterhood. o When she left she wanted her property back. o Held that there was a relationship of influence and that the circumstances called for explanation and therefore there was undue influence. However, she was barred from claiming due to her delay. Rebutting the presumption-Need to prove that the gift was the spontaneous act of the donor acting under his own free will. Shaik Allie Bin Omar. Roche v Sherrington o Plaintiff was a member of The Work of God. o After he quit, he brought an action claiming repayment for the sums he paid during membership. o He said he had been bound by vows of poverty, chastity and obedience. o Held that claim was arguable with sufficient relationship but it failed because the association was not a corporate body. Re Craig o Old man (84) employed a woman after his wife died to be a secretary and companion. o When he died, discovered that he made gifts of 25,000 to her. o This was challenged by his personal representatives. o Held that there was undue influence. Hewitt v First Plus National Group Ltd o Undue influence can take the form of misstating or suppressing information. o Here a wife mortgaged her interest in their home to pay for her husbands debts after he promised to be financially responsible.

o Husband failed to disclose that he was having an affair at the timeparties ended up divorcing. o Held that the husbands concealment of his affair was sufficient undue influence over his wife. Re Brocklehurst o Owner of a hall in Manchester. Wife and servant left him. o Struck up friendship with local garage man. o In his will it was found that he had given the shooting rights on his hall to the garage man which reduced value of estate. o Held that there was no undue influence as Brocklehurst had dominated the garage man. ASB Bank v Harlick o Daughter and son-in-law won $300,000 in New Zealand lotto. o Used money to expand business and later sought a bank loan. Bank required security. Father and Mother guaranteed the loan with mortgage on their house. o Business failed and bank wanted to exercise the sale of the house. o Held that there was no undue influence as this was a normal family relationship with the parents agreeing to assist their children. Hammond v Osborn o Disabled elderly man gave gifts of over 90% of his liquid wealth to a helpful neighbour. o Seemed that there was no consideration for his financial needs if he had lived and he would have been liable to Inland Revnue for a large sum. o The woman concealed these gifts and the personal representatives sought the money back. o Held that although the old woman was not guilty of bad behaviour, there was no rebuttal of presumption and so the amount was set aside.

Suretyship Transactions The bank may be bound where o The wrongdoer was acting as an agent of the bank, or o If the bank had notice of the wrongdoing or risk of wrongdoing. Agency-To be an agent there needs to be some direct communication with the guarantor or previous dealings, which imply that the wrongdoer is the agent. It is possible but rare. Notice Circumstances putting bank on inquiry Barclays Bank v OBrien (Husband and Wife)-Relationship with trust or manifest disadvantage. o Wife gave security over jointly owned home to secure loans owed to the husbands company. o Husband has misrepresented the amount of mortgage and then didnt pay. Bank wishes to enforce security.

o Held the wife was entitled to have transaction set aside because there is a real risk that the transaction will be misrepresented. o This added to the disadvantage to the wife put the bank on inquiry. o Unless the bank took reasonable steps to ensure the consent was properly attained-they are fixed to the husbands wrongdoing. Problems with this Doctrine of Notice o Mere relationship needed or proof of trust? o What other relationships can qualify? o How can the bank know if there is trust without making intrusive inquiries? o Sometimes a bank might be put on notice without any relationship simply by the fact of serious disadvantage.

Credit Lyonnais Bank Nederland v Burch o Employee asked by employer to give mortgage over her flat and an unlimited personal guarantee to a bank as security without being told of the companys debt by the bank. o Banks solicitors wrote to her advising that she seek independent advice which she ignored and went ahead with mortgage. o Company went into liquidation and bank sought to enforce its security. o Held that there was definitely a presumption of undue influence due to the relationship in question (fell within OBrien Principles) and the manifestly disadvantageous nature of it. o Also decided that bank had not taken enough steps. PRESENT LAW-Royal Bank of Scotland v Etridge (No. 2) o Lord Nicholls Bank is put on inquiry whenever the relationship is that of a husband and wife. There is no set list of possible relationships where one party may be subject to undue influence. Bank is put on inquiry in any case where the relationship between the surety and the debtor is noncommercial or if there is an unequal commercial relationship. The bank must then take reasonable steps to inform the guarantor the risks they are running. Bank is not put on inquiry where the money is advanced by way of a joint loan, unless the bank knows that it is not for joint purposes. CIBX Mortgages v Pitt-Here a joint borrowing for a normal and appropriate joint purpose-holiday home, but ended up being used by husband on stock market. Although the money ended up being abused and wrongly used by the husband, there was nothing to alert the bank to a risk of undue influence.

Banks do not have to follow the House of Lords! Steps to be taken by bank Bank must take reasonable steps to ensure itself that proper consent has been obtained. An example is insisting the wife attend a private meeting where she is told of her liability, warned of her risk and told to gain independent advice. Solicitors actions needed-Solicitor should go through the documents, advise on the risk involved, should point out the fact that the party has a choice or tell the party to wait for negotiation with the bank about the terms of the contract. This should take place in a face-to-face meeting without the other party. The solicitor can act for more than one party as long as the solicitor acts for the wife alone when giving her advice. If undue influence is probable, the bank must insist that independent advice is taken. However, the banks often didnt do this as they didnt want criticism for the advice they have given and simply told them to go and gain advice from a solicitor. Massey v Midland Bank o Held that it was enough for the bank to tell the wife to gain independent advice and be told subsequently that it had been given, unless the advice given was not satisfactory or if the bank did not give enough information. Royal Bank of Scotland v Etridge (No. 2) o Held that the wife was not truly being given independent advice as the solicitor simply made them sign an agreement. o The bank must take reasonable steps to ensure that the surety understands the nature and effect of the transaction and wishes to enter it. o Where the surety is required to take independent advice, the bank must communicate with her to ensure she knows the solicitor is acting for her and that thereafter she will be legally bound. o The husband should not be present at the meeting with the bank/solicitor. o If the bank has reason to suspect undue influence by the principle debtor, it must take reasonable steps to inform the solicitor or advise that the guarantor is advised by an independent solicitor. After this meeting, they must confirm the consequences with the wife once again. o If a solicitor gives bad advice then he can be personally liable with no effect to the bank. o If the bank does not do it, it is deemed to have notice of any claim that the transaction was produced by undue influence. o In this case the wife failed at the influence as there was no wrongdoing to impute to the bank.

N.B In suretyship case questions you must: 1. Consider whether there is undue influence or other wrongdoing by the principle debtor in relation to the guarantor. 2. Consider next whether the bank is put on inquiry and, if it is, whether it has take sufficient steps to protect the guarantor.
Midland Bank v Wallace o Wife alleged she gave a charge on advice from a solicitor to the bank. She believed that the advice was not sufficient and that the solicitor was instructed by the bank. o Held by the House of Lords that if she could prove this, it would show that the solicitor was not actually acting for her. Barclays Bank v Coleman o Here it appeared to the bank that the wife had received proper advice. o However, the advice was given by a legal executive as opposed to a solicitor. o Held that his made no difference and that the bank had carried out its duties properly. National Westminster Bank v Amin o Parents mortgaged house to fund a loan for their sons business. o Bank wrote to solicitors to make sure that the parents were aware to the terms of the mortgage. Solicitor claimed that he had. o Loan wasnt repaid, bank sought to enforce security. Mother claimed undue influence by her son. o Courts questioned whether the parents knew the solicitor was acting for them and also the fact that the bank knew the parents could not speak English and would be easy to exploit which should have been mentioned when instructing the solicitor.

Unconscionable Bargains
This protects people who for someone reason are at a particular disadvantage (poverty, sex, sickness, age etc which has been exploited by a stronger party)
Hart v OConnor o Lord Brightman-Bargain by an ignorant person acting without independent advice which cannot be shown to be a fair and reasonable transaction. It is victimisation which can consist of the active extortion of a benefit or the passive acceptance of a benefit in unconscionable circumstances. o Vendor of land suffered from senile dementia and sold a family farm to Hart on very good terms for Hart. Hart didnt know he was suffering from dementia and the court held he had no reason to expect it. o After his death his estate wished to claim but court held that he had been advised by his solicitor with the terms proposed by him and the defendant had no way of knowing. Nicholls v Jessup o Plaintiff owned land near main road and only access was narrow drive. It ran alongside defendants property. o Plaintiff proposed amalgamating the two drives together which was a huge advantage to the plaintiff-increased value of land by $45,000. It diminished the value of the defendants land. o Defendant then refused to perform agreement and plaintiff sued for specific performance. o First instance-defendant was ignorant about property, unintelligent etc. Held that plaintiff had not set out to take advantage but that you could find unconscionable bargain. o Court of Appeal held this finding and that the plaintiff should have taken account of the glaring disadvantage to the defendant and the fact she was ignorant. Gustaf v Mackfield o Director was terminally ill with cancer and agreed to purchase property for $12.3 million dollars. o He got valuations of property showing it was not worth this figure. o He then died and his executors sought to get out of it. They claimed the other party had taken advantage of him. o Held by the New Zealand Supreme Court that although his abilities were diminished and the other party was aware of the illness that it was not unconscionable as they didnt know

it prevented him from conducting his business properly and applied no pressure. The complaining party needs to show that: 1. He was in a position of disadvantage-poverty, sickness, age, sex, drunkenness, lack of education etc. a. Can also take place in commercial situations. Lease of a shopping centre. Lessors told lessees that they would grant extension on lease. Lessees had claims against lessors. Lessors said they would only grant the extension if the claims were dropped. Held that person is not in a position of disadvantage just because they are in an unequal bargaining power. 2. The stronger party knew or ought to have known this. 3. The stronger used his knowledge to victimise the complaining partyinadequacy of consideration, undue influence etc. Suretyship cases Here you must show unconscionability by the bank in a direct relationship by the guarantor. Very fact specific Commercial Bank of Australia v Amadio o Case covered guarantee by elderly, immigrant parents to secure their sons business borrowing. o Argued the bank had acted unconscionably as it knew the parents had not been fully informed of the state of their sons affairs. o Held that this claim succeeded. Portman v Dusangh o Old man who was illiterate and gave a 25 year mortgage over his house to guarantee a lending to the son. o Held that the building society had not exploited the father as they did not have to make decisions for the parent as to whether it was a wise thing to do. It was reasonable to assume it was done out of love and affection. o Conscience of the court must be shocked! Can be argued that there is an overlap with the notice principle found in undue influence. Not that the bank acted unconscionably but that they didnt take action when there was unconscionablity by one party.

Misrepresentation
An actionable misrepresentation is a statement of fact made by or on behalf of the representor to the representee that is false, material and induces the representee to enter into the contract. Introduction Purpose o To ensure fairness in the creation of contracts. o Provides remedies when contracts induced by misleading statements. Remedies 1. Rescission 2. Damages o Only available if the statement is one that the law recognises as a misrepresentation. All the elements of the definition must be found if the statement is to be misrepresentation. Rescission-If it was a misstatement then it is voidable-can be cancelled with retrospective effect at the option of the victim. Damages-Whenever a misrepresentation amounts to a tort, damages are available e.g. intentionally mislead through the tort of deceit or through negligence (Hedley Byrne) Also a statutory entitlement to damages through the Misrepresentation Act 1967, which provides a remedy where the law of tort would not. Remedy by right in breach of contract is damages whereas for misrepresentation it is rescission. It is much easier to prove misrepresentation under contract law (if the representation is part of the contract) than tort. Links with other doctrines 1. Mistake a. Two types of mistake-Bell v Lever Bros b. Nullifies consent-even though there was true consent, the law nullifies it (common mistake) e.g. in Bell-both agree to sell something which does not exist. The contract becomes void. c. Negatives consent-one which prevents parties from consenting. Typically a unilateral mistake e.g. mistake in the subject matter of the contract-Harthog. Only exceptionally voidable where the other party is aware of the mistake or if both parties make mistakes.

Semester 2

d. From Smith v Hughes we see that a person who enters into a contract with a unilateral mistake will remain bound by it if the other party was reasonably unaware that there was a mistake. e. The mistake must also be fundamental by concerning the contract itself. A mistake in value is not fundamental but a mistake in the terms, subject matter or identity of the other party is. f. DIFFERENCE-Doctrine of Misrepresentation covers all mistakes and dont take into account the additional requirements of the doctrine of mistake. Whenever there is a misrepresentation there is also a mistake but much easier to prove a misrepresentation and so the doctrine of mistake has basically been eclipsed. 2. Duress and Undue Influence a. If one person induces a family member to provide guarantee for a loan, this will often include a misrepresentation in relation to the loan. b. Misrepresentation will also sometimes award damages. 3. Contractual Warranty a. Dick Bentley Productions-Dealer sold Bentley car, representing that it had done 20,000 miles but in fact had done nearly 100,000. Through guarantees which undertake that a representation is true, the representation can be a term of the contract. b. Differences-The remedy of specific performance can order a party to fulfil the warranty but is no available for misrepresentation. c. The way of calculating damages is different in breach of contract than it is for misrepresentation. d. For an innocent misrepresentation you may be able to recover more if you show that it was a breach of warranty. Oscar Chess v Williams-private seller innocently misrepresented the year of his car when selling it to a dealer. Here it was treated as a mere misrepresentation as opposed to a term of the contract as he had no special knowledge. Actionable Misrepresentation 1. A Statement Misrepresentation needs to be eminent from the representor. Silence does not constitute a misrepresentation e.g. not informing someone about the fact that animals you are selling are infected. At common law there is no remedy for what you did not say. However, conduct can be used to cover a statement e.g. in selling animals to a market, you represent that they do not suffer any contagious disease. 2. Of Fact To be actionable the statement must be one of fact and not opinion, belief or intention. Opinion or Belief

o Difficult to distinguish opinion from fact due to things such as the fact that someone may claim to have an opinion even though they do not. o Bisset v Wilkinson-Vendor misstated its sheep-bearing capacity. Although he was sheep farmer the land had never been used for sheep farming. This was a mere belief. o Esso Petroleum v Mardon-Here there was a misrepresentation of fact as Esso lied about having an opinion which they didnt hold as they knew the closure of a road would damage sales. o It can also be actionable if someone holds themselves out as being an expert on the matter and the person places reasonable reliance on them. o Any expression of opinion must be founded on something. Economides v Commercial Assurance Co. Statements of Intention o A promise is not a representation as it is not a statement of existing fact. o Edgington v Fitzmaurice A borrows money on the basis that it intends to use it to improve its company. In fact its intention was to pay existing debts. Held that this was a misstatement of fact. o Wales v Wadham Wife promises not to remarry when she divorces husband who pays her maintenance. She does remarry but doesnt say it as she would lose the maintenance. Held that the husband could not rescind the contract as it was her then current intention not to remarry. Statements of Law o The general principle is that these are not misstatements as everyone can check the law themselves. o However, they can be if they are expected to know about the correct law such as landlords and tenants

3. Made By Or On Behalf Of The Representor To The Representee If you are induced by something false said by a 3rd party this does not give grounds to challenge a contract. A statement said by someone to a 3rd party that the contracting party overhears does not count. Exception-when the other party has notice of the misrepresentation of the 3rd party and fails to take reasonable steps. Identical principles from Etridge on undue influence. 4. False Generally straightforward to determine if something said is true or false when the fact stated has been clearly identified. 5. Materiality

It is an objective concept. The misrepresentation must be material except for fraudulent ones. It is material if a reasonable person would regard the misstated fact as relevant in deciding whether to enter the contract and on what terms. Unlikely that a court would conclude that a misstated fact which induced the representee would be regarded as irrelevant.

6. Inducement/Reliance/Causation Actionable only if the representee relies on the misrepresentation and is induced to enter into the contract as a result. Fraudulant misrepresentation o Edgington v Fitzmaurice-Only invested because of fraudulent representations as to the purpose of the money and a mistaken belief that the investment would be secured by a charge on the companys land. o Held that it was irrelevant that fraud was not the sole inducement. Suffices that the fraudulent statement was actively present in the mind of the misrepresentee. o Upheld in Barton v Armstrong Non-fraudulent misrepresentation o Assicurzaioni v Arab Insurance Group o To prove inducement the representee must show that but for the misrepresentation, he would not have entered into the contract on those terms. Demonstrating no inducement o Atwood v Small-if someone performs their own investigations they will be taken to have entered into the contract on the basis of their own investigations. o If the representee would still have concluded it on the same terms if it had known the truth, there is no inducement. JEB Fasteners v Marks, Bloom & Co. o Nocton v Ashburton-there is no duty to perform these checks. o Redgrave v Hurd Prospective buyer of practice queried the profitability of it. The seller said he was welcome to check the papers but the purchaser declined. Seller had in fact misrepresented the profitability. Held that just because the purchaser did not use the opportunity to check, it doesnt mean the other party can escape his liability. o This is the case for all misrepresentations (Standard Chartered Bank).

Remedies for Misrepresentation

1. Rescission Available for all forms of misrepresentation-fraudulent, negligent, innocent etc. By enforcing this, the representee decides that their consent to the contract should not be valid as it was flawed consent due to the misrepresentation. It is effective once he communicates the decision. The effect of this is retrospective and so it restores the parties to their pre-contractual situation. The contract is set aside and each party gives/gets back what has already been transferred. Redgrave v Hurd-Could rescind and get back the amount already paid and no longer had to pay the balance. The courts do not have to enforce this, it is up to the representee. If the court should confirm the invalidity of the contract then this is simply confirmation. Timing is crucial if the representee wishes to get back property transferred under the contract. This is because title to property can pass through a voidable contract and if a 3rd party then acquires it from the representor before the rescinding, he will become owner of it. o Car & Universal Finance v Caldwell Someone fraudulently induced to sell car in return for bad cheque. Buyer disappeared. Seller could not get hold of him but informed police and asked AA to recover car. The buyer then sold the car to a third party who knew nothing and bought it in good faith. Held that even though Caldwell hadnt managed to directly communicate the rescission, his actions were sufficient to amount to a rescission. Law Commission has advocated that the law be changed to accommodate innocent 3rd parties who act in good faith. Bars to Rescission 1. Unjust Enrichment The difficulty arises in non-monetary cases as the common law had imposed exact restoration of the precise thing received in the exact state. However, equity stepped in to change this strict law and allows substantial restoration to be sufficient. Erlanger v New Sombrero-Company bought mine under voidable contract but made profit from it and made it less valuable due to extraction of some ore. Restoration was not utterly impossible. Under common law, rescission was barred but was held that due to equity, they could rescind on condition that they return the mine and account to the seller any profits made from the mine. Equity is more willing to step in in the case of fraudulent representations rather than innocent ones. Smith Newcourt Securities-Court took the opinion that a buyer of shares cannot rescind if he has sold those shares, even if he is able to go into the

market and purchase similar shares and restore them. Lord BrowneWilkinson dissented saying you could rescind if you could provide the exact same shares with the same value. This is more flexible but shows that the law is still strict as you must restore the shares themselves and not some monetary equivalent. Trietal-Because the essential reason is to prevent unjust enrichment, the courts should become more flexible and should allow rescission whenever the representee is willing to provide the equivalent value of the thing acquired in money. However, representee can rescind if he can return the thing itself even if had lost its value over time o Armstrong v Jackson Stockbroker pretended to buy shares for client but in fact sold him his own shares. 5 years later client wanted to rescind it. Court held he could do so even though the value had declined dramatically as he was returning the very shares themselves.

2. 3rd Party Rights If a third party acquired rights to the goods that were a subject matter of the goods then it cannot be rescinded e.g. the 3rd party purchases goods in good faith from representor, the representee cannot rescind the contract. 3. Affirmation If the representee affirms the contract it can no longer be rescinded. There must be unequivocal act of affirmation such as statement or conduct and the representee must have knowledge of the misrepresentation and right to rescind. 4. Lapse of Time (Laches) If you fail to rescind within substantially reasonable time then you are held to have affirmed the contract even if the representee was ignorant as to the misrepresentation taking place. This helps to promote legal security and certainty. The period allowed depends on the facts of the individual case. Does not apply for fraudulent misrepresentations as rescission is never barred by lapse of time while the representee is ignorant to the misrepresentation. 5. Judicial Discretion Comes from the Misrepresentation Act s.2 (2). Allowed judges to refuse rescission in case of non-fraudulent misrepresentation if it would be equitable to do so in regard to the nature of the misrepresentation and the loss that would be caused to the parties. If used the court is directed to award damages instead. William Sindall v Cambridgeshire CC o Purchaser bought land off council for 5 million.

o 2 years later the claimant discovered that there was a sewer running underneath it which would prevent redevelopment unless he incurred a small cost to re-route it. The value of land had also gone down by a lot. o They wanted to rescind the contract to put them back in better position. Williams could argue there was misrepresentation. o Held that there was no representation. But, according to Lord Hoffman, this was a type of case where the courts should exercise discretion and prevent rescission as the misrepresentation was minor and the losses suffered by claimant as a result were much smaller than the loss that would be suffered by the representor if it were rescinded. 2. Damages Four Possibilities 1. Warranty: breach of contract. Collateral Warranty-court trying to bring a representation into the contract. 2. Indemnity for wasted expenses. 3. Damages for common tort law. 4. Misrepresentation Act 1967: s 2 (1); s 2 (2). o All of these possibilities except for the breach of contract/s 2 (2) can co-exist with rescission. 1. Indemnity o Years ago the only way to claim compensation for misrepresentation that was not fraudulent was through this. o It covers all expenses incurred by reason of entering into the contract. o Whittington v Seale-Hayne Poultry farmer took lease of premises but it turned out they couldnt as they werent in proper state and so rescinded the contract. By way of indemnity they could recover the amount spent on local taxes as well as money they had to spend under law to improve the drainage. 2. Damages at common law o Deceit Derry v Peek-to succeed the representee must prove that the representor made the statement either knowing that it was false or without belief in its truth or recklessly. No limit to the damages. o Negligence Hedley Byrne v Heller-Availability of damages turns on whether the representor owed a duty of care and breached it. (foreseeability, reliance, proximity) o The basic mode of calculating damages is the same for all torts. It is to place the victim in the net financial position that he would have been in if the tort had not been committed.

o However, the remoteness rule does not count for deceit cases and so can be compensated for all losses, even those that were not reasonably foreseeable. 3. Misrepresentation Act 1967 o It does not abolish or reform common law actions but provides parallel basis which overlaps with and goes further than the common law. It is much easier to claim under this act. o However the act only covers misrepresentation by the parties to the contract itself and only where a contract actually came into being. o Section 2 (1) Only available if misrepresentation is made by another party to the contract. Reverses the burden of proof to the representor after the representee has shown a misrepresentation caused them loss. Makes someone liable for misrepresentation unless he had reasonable grounds to believe, and did so up until the time the contract was made, that the representation was true. It also makes fraud approach to damages available to all cases, which is more advantageous to the claimant. Removed need to look at Hedley-Byrne. o Howard Marine v Ogden One party claimed damages where it could not be proven that the misrepresentation had been due to the carelessness of the representor or whether it was an error that was reasonable to make. Under the common law the representee would have lost but under the act he succeeded as the burden of proof had switched. o The principle of contributory negligence does not usually count for deceit cases. o Royscot Trust v Rogerson-Held that the measure of damages recoverable for innocent misrepresentation under s.2(1) of the 1967 Act was the measure for fraudulent misrepresentation rather than for negligence. They read the words so liable in the statute as meaning that they had to award these damages. Section 2 (2)-covers innocent misrepresentation Allows the awarding of damages in lieu of rescissiondiscretionary. William Sindall v Cambridgeshire County Council. o Effect of bars to rescission Government of Zanzibar v British Aerospace Held that because s 2 (2) damages are an alternative to rescission, the court only has the power to award damages if it has power to award rescission. Here it didnt due to delay.

This seems like the correct answer even though there is lack of clarity. o Measure of Damages The act recognises the possibility of concurrent awards under s 2 (1) and s 2 (2). However, the measure of damages is different between the two. Damages under s 2 (2)-aim to place the representee in the same position as if the contract had been rescinded. Suggested by Lord Hoffman/Evans that this is not correct.

Introduction There are both express terms and implied terms. We need to distinguish: o Whether a particular provision is part of a contract (Incorporation). o What does a term mean? (Construction)-always objective (reasonable person). Parol Evidence Rule-Where a contract has been expressed in writing, evidence cannot be induced to add to, vary or contradict the written document. Courts do not have common law powers to cut down on unfairness in contract terms. However, they can exert an indirect influence by deciding whether or not a particular term is incorporated into the contract. However, nowadays there are direct controls via statute. Incorporation still plays a big role such as in business to business contracts. Incorporation of Terms Basic principle: o Whether a term is incorporated into a contract depends on the agreement of the parties. Factors: 1. Timing-term must become known before the contract is concluded. Olley v Marlborough Court o You could only see a notice once you had concluded a contract with a hotel and entered your hotel room. Held the term was not incorporated. 2. Signature-someone who signs a contract is bound to its terms, even if he has not read them. Must be a document that a reasonable person would expect to be a contract. LEstrange v Graucob o Owner of caf bought vending machine. Signed sales agreement without reading it which contained exemption clause for any defects in the machine. Held that she was bound by this. o Exceptions If a party, through no fault of their own, did not have any real understanding (illness, education etc) of the document and was not given a full explanation. Induced to sign contract due to misrepresentation.

Ascertaining the Terms of a Contract

Grogan v Robin Meredith Plant Hire o Here a hirer signed time sheets every week with terms in it. Held that no reasonable person would expect the time sheets to contain contractual terms. Tilden Rent-A-Car v Clendenning (Canada) o Someone hired a car and signed very quickly a long agreement which was difficult to read. o The contract contained unfavourable terms. o Held that in these circumstances, the signature rule does not lead the person to being bound. English law has not yet taken this view.

3. Notice (ticket cases)-English Courts require that the enforcing party took steps to bring the existence of a term to the notice of the other party. A lot of issues with the clauses on the back of tickets. Parker v South Eastern Railway o Court of Appeal held that clauses on the back of tickets did bind customers but because there was no signature, the enforcing party would have to bring other evidence to show that the customer assented to the terms. The company would have to show that they had taken reasonable steps to bring the conditions to the attention of the customers. Thornton v Shoe Lane Parking o The entrance ticket to a car park contained a term which sought to exempt the company from liability from any personal injury. o Customer argued that this particular clause was unusual and there was no reason to expect such a clause. o Held by the Court of Appeal (Denning) that where an unusual restriction in that class of contract is relied on, then it must be shown that it had been fairly brought to the notice of the other party. Interfoto v Stiletto Visual Programmes o Plaintiff was a company which lent out photographic transparencies. o Defendant borrowed some of them which were delivered to him. The delivery note contained several printed terms, one of which said that if the item was not returned within 14 days that the borrower had to pay a holding fee of 5 a day plus VAT. o Company returned the items two weeks late and was asked to pay the holding fee but refused and were sued. o Court of Appeal held that the term had not been incorporated into the contract as they had not brought this unreasonable term to the attention of the defendants. Unlike Thornton it did not contain an exemption clause and so this case applies generally. Lord Justice Dillon/Bingham said that Thornton applies

whenever there is a particularly unreasonable or harsh consequence. AEG v Logic Resource o Here the clause in question dealt with a buyers remedies for defective goods, saying the buyers only remedy was repair and that the cost of returning the goods was at the expensive of the buyer. o Held by majority in Court of Appeal that it wasnt incorporated because it was an unusual and onerous term. To pursue fairness in this indirect way the Courts act in an underhanded way and produce artificial reasoning with inconsistent outcomes. This creates a lot of uncertainty in the law. Generally notices at the back of documents, covered by stamps will not be incorporated into a contract. It is also based upon how unusual the term is.

4. Incorporation by Reference-It is not a requirement that all terms must appear in a single document. It is possible that a clause in question is merely referred to in the main document. Implication of Terms Inserting terms that have not been expressly agreed into contracts. Important to distinguish between terms that are implied in fact (implicit in the express agreement) and those that are implied in law (inserted in virtue of the operation of the law). 1. Terms implied in fact Southern Foundries v Shirlaw-Officious bystander o The test is whether something is so obvious that it goes without saying so that if an officious bystander had suggested some express provision for it, they would reply oh, of course. The Moorcock-Business efficacy o The test here is whether an inclusion of the term would make the contract more efficient from a business point of view, whether it is necessary. o This case dealt with a river bed adjoining a jetty which caused damage to a vessel and it went along with business efficacy for the defendants to have impliedly taken responsibility for this. Attorney-General of Belize v Belize Telecom o Lord Hoffman said it is dangerous to treat the previous tests as if they have a life of their own. They are simply different ways for judges to express the same idea. The one question is what the instrument would reasonably be understood to mean. 2. Terms implied in common law

Differences to implications in fact-Here the terms are implied even if none of the parties would have agreed to it. It focuses on the class of contract it is. Liverpool County Council v Irwin o Things were not properly maintained in a block of council flats and so the tenants withheld their rents and were subsequently sued. o They argued that the council was in breach of their contractual duties to maintain the common parts of the flats. o However, the written tenancy agreement said nothing of this. o Court of Appeal held that there was no such implied term in the contract because this implied term would not pass the officious bystander test or the business efficacy test. o House of Lords held that the particular clause was to be implied into the contract. They agreed it would not be implied in fact. However, they held that despite this, the term should be implied in law. o The question of whether it should be implied by law came down to the question of whether a clause of that type was necessary in a contract of that sort i.e. whether all tenancy agreements in multi-story flats needed a clause of this kind. Scally v Southern Health & Social Services Board o Complaint by doctors who made a compulsory contribution to a pension fund. They could make additional payments within 12 months of starting the job but they were never informed expressly by their employers of this. o They proved that had they been told they would have taken up the opportunity and they argued that there was an implied term in the contract, o Held that there was an implied term by the House of Lords-a lot more relaxed than Liverpool CC as there was no necessity for this term. It also shows that the Courts focus more on a specific contract as opposed to a broad type of contract. Terms implied by custom o Where it is the custom of the market, trade or locality to include a term of that type. o Hutton v Warren It was said that the parties contract with reference to the usage of the customs of the trade etc. The custom must be: Certain Notorious (well known all around) Recognised as binding Reasonable Not contradict the express terms. Terms implied by statute o Here a statutory provision inserts a term into a specified type of contract e.g Sale of Goods Act 1979

The Validity of Contractual Terms


Introduction A judge will normally enforce a contract even if he feels a contract was unwise, foolish or unfair-they do not police the fairness or the content of the substance of contracts. However the Courts have direct ways of policing the terms of contract. Originally, under common law the exemption clauses were enforceable due to freedom of contract but as standard form contracts became more prominent which often created unfairness the country needed statutory intervention. If a date/time is stipulated in a contract, the court will take it very seriously. Common Law Controls Penalty Clauses (Liquidated damages clauses) o Say that a contracting party will have to pay a certain sum or forfeit something if they breach the contract. o There is a risk of unfairness that the penalty sum may be out of proportion to the loss suffered and so the Courts will only enforce it if it is a genuine estimate of what the loss might be. Restraint of Trade Clauses o Here, one party to a contract agrees that after the expiry of a contract, he will not compete with the other party e.g employee estate agent after resigning from his job. o Again there is a risk of unfairness and so the Courts exercise control over the clauses and will only enforce the clause if it is reasonable. Exemption Clauses o Here, someone seeks to limit or exclude liability in the event of a breach of contract. o Fundamental Breach Liability for a fundamental breach of a contract cannot be excluded. Photo Production v Securicor-House of Lords ended this restriction, however, there was now the statutory protection by the Unfair Contract Terms Act 1977 o Strict Construction The Courts will construe exemption clauses narrowly and are unlikely to interpret an exemption clause as covering a fundamental breach. Statutory Controls The following two statutory controls slightly overlap. They use different terminology and techniques. Differences

o UCTA applies to consumer, commercial contracts and notices that seek to exclude liability in tort whereas the regulations apply to consumer contracts only. o UCTA covers exclusion and limitation of liability clauses whereas the regulations apply to any other term except for definition of the main subject matter of the contract or the price. o UCTA makes certain clauses invalid whereas the regulations simply subject them to a fairness test. o UCTA test turns on notion of reasonableness whereas the regulations turns on fairness. o UCTA contains guidelines for the application of the test whereas the regulations contain a Grey List which are a list of clauses, presumed to be unfair. o Under UCTA, the burden of proof of showing reasonableness is on the party seeking to rely on the clause whereas in the regulations, the burden of proof is put on the consumer. o UCTA applies to terms whether it is in a standard form contract or it was an individually negotiated provision whereas the regulations only applied to standard form. o UCTA does not apply to certain categories of contract whereas the regulations apply to all types of consumer contracts. o UCTA applies only between the two parties to the contract whereas the regulations empower bodies to take action to prevent the use of unfair terms. o UCTA has a separate provision for Scotland whereas the regulations apply to the UK as a whole. 1. Unfair Contract Terms Act 1977 (UCTA) First attempt to protect people from contract terms that are to the advantage of one party only. It also tackled exemption clauses by either declaring them invalid or subjecting others to a reasonableness test. Scope of Act o Unlike the regulations (below) it is not restricted to consumer contracts-also covers contracts between businesses. Needs to be at least one business party. o Section 26-International supply of goods contracts are not covered. o Schedule 1-UCTA does not apply to insurance contracts, contracts for the creation or transfer of interests in land or contracts for the creation or transfer of interests in intellectual property. o Covers only one specific type of term-term which seek to exclude or limit liability (exemption clauses). o Manner of control differs between different types of contractual obligations as each obligation is covered in a separate section. Provisions o Operative Provisions-set out how UCTAs controls operate in respect of different types of contractual obligations.

o General Provisions-these affect the operations of the operative provisions. They specific the test for unreasonableness (Section 11) and also a definition of exemption clauses (Section 13). Section 6 o When it applies-Breach of implied terms under the Sale of Goods Act 1979/Hire Purchase Act and Implied Terms Act 1973. o 6 (4)-This control also applies when the liability is not business liability whereas Sections 2-7 only applies to business liability. o What sorts of clauses-Section 13-Gives the list of exemption clauses that are controlled by Section 6-quite wide ranging e.g. clauses which make it more difficult for people to enforce a particular contractual remedy. A contractual term which seeks to prevent an obligation or liability from arising is also covered. o What control-6 (1)-liability from Section 12 of the Sale of Goods Act cannot be excluded or restricted. 6 (2)-where the buyer is a consumer, liability for these breaches or other implied terms cannot be excluded or restricted. 6 (3)-if the buyer does not deal as a consumer then the liability can be excluded only as long as the contractual term satisfies the test of reasonableness. o Who is a consumer for the purpose of this? R & B Customs Brokers v United Dominions Trust Claimants purchased car for director. They sued as were unhappy but defendants relied on exclusion clause. Court held that the company dealt as a consumer for the purposes of the act as the contract was not integral to the nature of the business and did not form part of the regular course of business. They were not in the business of cars. (Under the regulations (below), only a natural person can be a consumer). o Section 11-sets out what the reasonableness test is. 11 (2)-refers us to Schedule 2-sets out the factors when deciding the term in question is reasonable or not for the purposes of Section 6. o Reasonableness usually harder to prove when the contract is business-consumer rather than business-business. Section 2 o When it applies-to negligence liability. Only applies when there are attempts to exclude or restrict liability for negligence where that liability is a form of business liability (liability by things done or to be done by people in the course of business or from the occupation of premises used for business purposes of the occupier). Negligence-covers tort and contract as well as obligations from the Occupiers liability Act. o What sort of clauses-turn to Section 13. o What control is exercised Liability for death or personal injury can never be excluded or restricted. 2 (1)

Any other form of negligence can only be excluded or restricted if they satisfy the requirement of reasonableness. (Section 11/Schedule 2).

Section 3 o When it applies All attempts to exclude or restrict liability for breach of contract. Overlap with Section 2 and 6 (Section 3 is merely a safety net provision). Either when party tries to restrict liability for substantially different performance or no performance at all. Only applies if one party either deals as consumer or deals on the others written standard terms of business (changing of clauses may render a standard from contract as nonstandard)-needs to be more than 1 term. McCrone v Boots Farm Sales-Held that the Act also covers contracts which are partly written and partly oral as well as partly on fixed terms and party negotiated terms. A standard term of business created by a third party, such as a trade association, is only covered if that party usually uses that form. (British Fermentation Products Ltd). o What sort of clauses-Turn to Section 13 (1)-clear that this does not apply to Section 3. Therefore, the controls do not apply to provisions which seek to prevent a liability from arising in the first place. o Control-applies reasonableness as a test. It also controls any contractual term which authorises one party to change the service that is provided that was decided upon originally as well as someone trying to render no performance at all in respect of his contractual obligations. Section 8 (Misrepresentation Act, s 3) o When applicable-when there is fraudulent misrepresentation there is no need to consult the Act, simply covered in common law. o What sort of clauses-Turn to Section 13. Just like in Section 3, controls do not apply to provisions which seek to prevent a liability from arising in the first place. o Control-Always the reasonableness test. Entire Agreement Clauses o Deepak v Imperial Chemical Industries Had a term which claimed that the contract was the entire agreement. Seeks to prevent someone arguing that the other party is liable for certain kinds of breaches. Held that provisions of this kind are not covered by UCTA because the clause is only to prevent claims for breaches of collateral warranty. It does not restrict or

exclude a liability that exists. However, the claimant could still bring a claim for misrepresentation. o However, they can fall under UCTA if there are additional elements following it-no other undertakings of any description have been given. No Reliance Clauses o This can potentially affect someones ability to bring a claim for misrepresentation, saying that no statement or representation by either party was relied upon by the other when agreeing to enter into the contract, o Grimstead v McGarrigan o Watford Electronics o Basis for arguing that clauses of this kind are covered by UCTA-Section 13 (1c) but there is no clear authority on the matter. Courts will normally apply a rule of strict construction in attempts to limit liability (interpret the clauses as narrowly as possible to restrict their effects and impact)-Cremdean Properties v Nash.

Common Law Relationship 1. UCTA does not deal with incorporation. IN A QUESTION ALWAYS LOOK AT WHETHER A TERM HAS BEEN INCORPORATED INTO A CONTRACT UNDER COMMON LAW BEFORE TURNING TO UCTA. 2. When UCTA does not apply but you are still dealing with an exclusion clause. George Mitchell v Finney Lock Seeds P ordered late cabbage seed from D who were seed merchants. The conditions of sale limited D's liability to the replacement or refund of the value of the seed. The seed proved utterly wrong for the purposes it was required. Held, dismissing D's appeal, that although the conditions did limit their liability under contract, reliance on the conditions would not be fair and reasonable within the meaning of the Sale of Goods Act. Stewart Gill v Horatio Myer A term in a contract which includes a prohibition on a payment or credit being set off against the price claimed is prima facie unreasonable. When considering reasonableness, the whole of the term had to be considered, not just that part relied upon by one party. Here the set off clause had the effect of an exclusion clause even though it was not one-courts made it come under UCTA through Section 12. Stevenson v Rogers Defendant sold a fishing boat to buy another, the very thing that his business needed to survive.

Section 3 of the 1973 Act, re-enacted by s.14(2) of the 1979 Act, was intended to widen the protection afforded to a purchaser and, given the removal of the requirement that the seller deal in the type of goods sold, there was no justification for the re-introduction of an implied and indefinable qualification on the apparently wide scope and purpose of s.14(2). Accordingly, the sale of the "Jelle" was "in the course of a business" and was subject to an implied term as to merchantable quality.

3. Unfair Terms in Consumer Contracts Regulations 1999 Based on an EU directive. This replaced regulations from 1994. Growing awareness of need to protect consumers who were exploited by businesses who imposed unfair contract terms and to broaden the scope from UCTA. Basic Effect-Regulation 5-protect unfair contract terms which are to the detriment of a consumer. Scope of Regulation o Regulation 4 (1)-applies to contracts between a seller or a supplier (natural or legal person acting for his trade) and a consumer (natural person not acting for his trade). Immune Terms o Basic approach is to subject all terms to the regulations but to exclude certain types of terms. o Regulation 6 (2)-anything that defines subject matter or lays down the price is usually immune as long as it is drafted in plain, intelligible language. Treital-the regulations are not there to monitor quality or price control. D-G Fair Trading v First National Bank Decided under the 1994 regulations which had identical wording. Dealt with a challenge to a contractual clause for a loan which said that if the borrower ended up in default, then extra interest had to be paid until the loan was fully repaid. Court held that this provision was not part of the price, was not immune from challenge and so could be subjected to the fairness test. Bairstow Eves London Central v Smith Contract between seller and estate agent which stipulated that a commission fee of 1.5% if the client paid within 10 days or 3% if the customer paid after this time. Held that Regulation 5 could be applied and it was unfair to charge someone double the price

for not paying quickly enough and that it was down to a value judgment. Office of Fair Trading v Abbey National This dealt with whether the regulations could challenge the charges that high street banks levy on current account holders for unauthorised overdrafts. Supreme Court held that the provisions were immune from challenge as they were part of the price because they were a very important part of their charging structure. However, if fees only play a marginal role, they may not be part of the price and may be challengeable. o Regulation 5 (1)-individually negotiated terms are also immune. The regulations are aimed at standard form contracts. o Regulations 5 (2,3 &4) define what counts as individually negotiated. Also shows that the burden of proof is for the seller/supplier. Test for unfairness o Regulation 5 (1)-terms are regarded as unfair if, contrary to the requirements of good faith, causes a significance imbalance in the parties rights and obligations to the detriment of the consumer. o 5 (5)-Schedule 2 contains an indicative list of terms which are unfair (though the presumption can be rebutted) e.g businesses trying to charge high penalties to consumers who fail to fulfil their obligations or clauses which enable a seller to alter the terms of a contract with a valid reason. o Regulation 6 (1)-unfairness takes into account the nature of the goods or services and referring to all the circumstances attending the conclusion of the contract and also the balance of the contract as a whole. o D-G Fair Trading v First National Bank (above) The high interest charge on defaulting a loan contradicted the Consumer Credit Act whereby the Court could make a time order which enabled debtors to pay back credit over a period of time and to also vary the terms of the original contract so that default interest does not have to be paid for the whole period of time. In practice, where a time order is made by the Court the default interest keeps on running as the debtors are unaware that of the above provision on preventing default interest being constantly paid. D-G argued that debtors who cannot pay will be suddenly surprised with the obligation to pay a much higher sum than anticipated even though they have been given an extension. Court of Appeal-the clause was unfair, led to unfair surprise.

House of Lords-the clause was not unfair and all the clause did was ensure that the bank had to go to Court to get judgment. If there was unfairness that was simply due to people not knowing their rights properly. o Bankers Insurance Co v South South went on holiday and took out an insurance policy. He was involved in jet ski accident and injured a third party. 3 years later the third party sued. South was only informed several months after he had been sued. However, the insurance policy stated that the payment of claims was dependant on the insured reporting in writing as soon as possible, full details of any incident, which may result in a claim and also to forward all documents immediately upon receipt. Insurers refused to pay. South challenged this as well as another provision which excluded cover for motorised waterborne craft. Court held that the motorised waterborne craft did include jet skis and that the clause was a definition of the subject matter of the contract and therefore was immune from challenge under Regulation 6 (2). However, the first provision dealt with neither subject matter nor price and so were not immune to challenge. Held that this provision was unfair as sometimes a breach of the conditions such as late notification may not cause difficulties for the insurer. Consequences of finding unfairness o Regulation 8-An unfair term will not be binding on the consumer. However, the contract will continue to bind the parties if it can continue without the existence of the unfair term. o South-Court took the view that the regulation should be interpreted that the Courts can read down a particular provision with the effect that a Court can hold a provision remains in force to the extent that it was fair. The Court in South held that the provision was only not enforceable to the extent that it was unfair. Therefore it could be enforced to the extent that it was fair. The Court changed the clause to make a much narrower application of it and saved the clause from being rendered completely ineffectual. Enforcement of Regulations o Regulations 10-12-Enforceable in not only individual litigation but also through litigation conducted by the Direct General of Fair Trading. This is a more consumer friendly way of challenging unfair contract terms. Procedure 1. Are parties consumer and seller? 2. Was the term individually negotiated? 3. Does the term define the subject matter or price? 4. Whether the term in question is unfair or not

Common Mistake and Frustration


Introduction to Mistake Mistake o Agreement mistake-unable to say that parties reached agreement. This negatives consent. o Substance mistake-(Common Mistake)-parties are in agreement but mistaken about a vital characteristic of the contract. This nullifies consent. Objective approach abandoned when you know about someone elses mistake-not allowed to take advantage. Most mistakes are induced by some misrepresentation by another party. Agreement Mistake Prevents a genuine agreement being reached between the parties. General rule in unilateral mistakes (Smith v Hughes): o If, whatever a mans real intention may be, he so conducts himself that a reasonable man would believe he was assenting to the terms proposed by the other party, he would be equally bound as if he had intended to agree-Blackburn J Exceptions (only negative consent when): o If the mistake as to the terms of the contract are known to the other party-Harthog-price per piece/price per pound. There are about 3 pieces in a pound. The other party must have known there was a mistake. o If the parties are at cross-purposes as to the subject matter of the contract, each having made an understandable mistakeScriven Brothers v Hindley-buyer at auction believed he is bidding for a hemp when it in fact consists of a hemp and tow. o Where to the knowledge of A, B is mistaken as to As identity, thinking he is C-Cundy v Lindsay. In all of the above the mistake must be fundamental-Smith v Hughesno ground for relief over a simple mistake as to the nature of the oats. o If the seller knew the buyer was labouring under a mistake as to the subject matter he still owes no relief. o However, if the buyer makes a mistake as to what the seller was promising and the seller knew this, then this does give rise to relief. Misrepresentation, however, covers all mistakes and is much more widely used. However, it cannot be used when there is silence or when goods have been transferred to a good faith 3rd party as misrepresentation makes it voidable whereas mistake makes it void. Substance Mistake Bell v Lever Brothers

o Bell was director of company which was a subsidiary of Lever Brothers. When LB reorganised its business it agreed to pay Bell 30,000 to terminate his contract o The then found out that Bell had been in breach of his duty of fidelity to the company which meant he could have been fired anyway and refused to make the payments. o House of Lords held that Lever Brothers were bound by the severance payment due to the fact that they obtained the same release of the contract that they had always intended for. o Lord Atkin-Two situations where contract will be void for common mistake Sale of something which ceases to exist. (Res Extincta) Sale of something to a buyer who was already to owner. (Res Sua) o He also formulates what makes contract void in mistake of quality Must be one which makes the thing without the quality essentially different from what it was believed to be. An example is a lease to an unfurnished house which is believed to be habitable but which has a defect which leaves it uninhabitable or a painting thought to be an old master which is not. This is still binding as the thing is not essentially different. Equitable Doctrine (Old Law) o Common mistake which was insufficiently fundamental to be covered by Bell could make a contract voidable in equity-Solle and Butcher o Court of Appeal rejected the whole doctrine in Great Peace Shipping v Salvage. Where is Bell satisfied? o Associated Japanese Bank v Credit du Nord A man wanted to borrow money from bank. It took the form of a sale and lease-back agreement. The bank wanted further security and Credit du Nord agreed to give guarantee. The man defaulted on payment and it turned out that the machines the man said he was selling and leasing back did not exist. Held that the guarantee was void because one based on an unsecured transaction was fundamentally different from one based on a secured transaction. o Couturier v Hastie Simple reason the claim failed is because the buyer did not receive the goods. There was no mention of the terms void or mistake in the case itself and so is not really related to Bell. However, it could also be that the contract was void because the product did not exist-res extincta.

Section 6 of Sale of Goods Act 1979-where there is a contract for the sale of specific goods, and the goods without knowledge of the seller have perished at the time when the contract is made, the contract is void. McRae v Commonwealth Disposals Commission (Australia) Commission sold to McRae who was a scrap seller, a tanker. However, this did not exist and the place it was described as being in did not exist either. Held that it was not a case of res extincta but that there was no rule that a contract for the sale of goods that does not exist is automatically void-all depends on the construction of the contract. Held that the Commission was in breach of contract. Nicholson and Venn B bought table linen described as property of King Charles I. Turns out it dates from 150 years later. Held that they were entitled to damages. Scott v Coulson B buys a life insurance policy on the life of X, both B and S believing X to be alive In fact, X is dead and the policy has matured. Held that S was entitled to have the transaction set aside. Sherwood v Walker S sells cow to B, both believing it to be infertile. It actually turns out to be fertile. As a breeding cow it is worth more than 10 times the amount he paid for it. Held that the mistake went to the core of the agreement and that therefore the contract was unenforceable.

Introduction to Frustration Very limited principle with the courts only using it if they cannot find a breach of contract by any means. No general principle that liability for breach of contract depends on fault. National Carriers v Panalpina-tenants had 10 year lease. Local authority closed road for 18 months which closed access to land. Held that law of frustration applied to leases but that the 18 month interruption was not sufficiently serious. Unexpected expense unprofitability or difficulty in supply does not constitute frustration. Effect of Frustration o Contract is automatically terminated by law. The parties have no choice in the matter. What is Frustration? Taylor v Caldwell o Defendant hired out hall for concerts which was accidently burned down.

o Held that the burning of the hall excused both parties from their contractual duties based upon an implied term in the contract. Davis Contractors v Fareham Urban District Council o Contractor contracted to do work. But because of shortage of labour and resources, it took much longer than contemplated and became unprofitable. o He said that the delay had frustrated the contract and so deserved to be paid on the basis of the work completed. o However, the contract contained various provisions on delay. o Held that he lost his claim. Jackson v Union Marine Insurance o Contract to carry cargo of rails to America. The ship grounded soon after departure. The voyage had to be suspended though it could travel months later. o Court held that the contract was frustrated. Krell v Henry o Defendant contracted to pay money for use of claimants flat for two days with the sole purpose of watching Coronation procession. o Coronation ended up being cancelled and the claimant sued for the balance of the money. o Held that that cancelling of the Coronation frustrated the contract as it was the foundation of the contract itself and also due to the purpose behind the contract. Said to be different from a contract to hire a cab to Epsom on Derby Day as it would not be frustrated by the cancellation of the Derby and there was no special reason for choosing a particular cab. o Frustrating event must make the situation radically different. Herne Bay Steamboat Co v Hutton o This dealt with chartering of a boat to watch the naval review. However, due to Edward VIIs illness he could not show up. o Court held that they still had to pay for the boat and this was not a frustrating event. Some arguments for this include the fact that the purpose was to see the fleet (which was still there) as opposed to seeing the King. Also thought that its purpose was business, to hire the boat out to people. Griffith v Brymer o Due to the fact that the procession had been cancelled before the contract was signed, it was held to be void for mistake. o It was upheld that frustration can only take place after the formation of the contract. Noblee Thorl o Sale of nuts on standard CIF terms (all in price for transport and insurance). o Before shipment, the Suez Canal was closed and voyage would end up being much longer. o Sellers refused to perform contract, saying it was frustrated. o House of Lords held that the difference in the voyages was not fundamental enough to frustrate the contract. Said it might be

different if the cargo was perishable. There was also no express term in the contract saying the voyage would be via the Canal but even if there was the contract would still not have been frustrated.

The Terms of the Contract and the Knowledge of the Parties Risk-Saying that something is at the buyers risk in the contract means that a contract wont be frustrated if the risk materialises e.g ship sinking. Frustration of Illegality-frustrated regardless of express clauses in the contract. Metropolitan Water Board v Dick o Contract to construct reservoir within years. Expressly given extension of time for delay, however occasioned. o Government ordered contractors to stop work and do more important work during war times which ended two years later. o House of Lords held that the clause was never intended to cover a cessation of work indefinitely. Therefore, the contract was frustrated. Express clauses of this nature are known as force majeure clausesfrustrates contract due to events beyond control of the parties. Foreseeability-The general rule is that an event which was foreseeable at the time of entry into the contract does not operate to frustrate the contract. The less that an even is foreseeable, the more likely it is to be a factor which would lead to the frustration of a contract. Risk foreseeable but contract contains no provision: o Ocean Tramp Tankers Charter to carry iron though Suez Canal. When contract was made, the possibility of the Canals closure had already been raised. Held that frustration did not apply as firstly, the voyage they had to undertake was not sufficiently different and secondly, the reason the ship did not get through the Canal was due to delay by the ship-owner-self induced frustration. Lord Denning obiter-fact that the closure was foreseen would not prevent frustration from operating. The only essential thing is that the party made no provision for it in the contract. When is the Contract Frustrated? Cases often brought many years after the frustrating event. Immediate action may need to be taken. If there is an event which leads people to expect frustration but things turn out unexpectedly well: o Embiricos v Sydney Reid

Ship chartered to carry goods. However, conflict came about between Turkey and Greece and charterers thought the ship would be stuck in the Black Sea. However, the Turks eventually allowed 7 days for ships to pass though. Court of Appeal still held that the contract was frustrated with the view to reasonable businessmen at the time, not hindsight.

Self-Induced Frustration You cannot take advantage of your own wrong. It is effectively breach of contract and includes negligence in commercial context (but not always in non-commercial) Joseph Constantine o Claimant chartered ship but before loading began, there was an explosion which caused such a long delay which frustrated the contract but the cause was unknown. o The charterers sued for failure to load. o House of Lords held that the contract was frustrated by the delay unless the charterers showed that it was the fault of the owners. Choosing between contracts o Maritime National Fish v Ocean Trawlers (Canada) The respondents were owners and the appellants were charterers of a steam trawler which was fitted with, and could operate as a trawler only with, an otter trawl. By the charterparty the vessel could be used only in the fishing industry Canadian statute, which was applicable, made it an offence to leave a Canadian port with intent to fish with a vessel using an otter trawl, except under licence from the Minister. In March, 1933, the appellants applied to the Minister for licences for five trawlers which they were operating. The Minister intimated that only three licences would be granted, and requested the appellants to name the three trawlers in respect of which the three licences should be granted. The appellants named three trawlers, excluding the trawler now in question, and accordingly licences were granted for those three only. The appellants thereupon claimed that they were no longer bound by the charterparty, and to an action claiming the charter hire pleaded that the charterparty had become impossible of performance and their obligations under it ended: Held, that there had been no frustration of the charterparty, as the absence of a licence was due to the

election of the appellants, who remained liable for the hire. o The Super Servant Two Two specialised ships owned for carrying oil rigs. Contracted to carry rig for claimants given them the option between their two ships. Before performances, the Super Servant Two sank and it was not known whether it was due to negligence. The Court of Appeal held that the frustration was selfinduced as the defendant exercised a choice. Consequences of Frustration It automatically terminates the contract as to the future. This can give rise to some problems: One party pays in advance o Fibrosa Sellers sold machines to buyers. Buyers paid 1,000 in advance. Before the delivery, the Germans had invaded Poland. The contract was therefore terminated by frustration. The buyers wished to reclaim the money paid in advance. Orthodox view that case must fail. In Chandler v Webster, the Court held that advance payments could not be recovered because it a claim in restitution and only rested on consideration being present. House of Lords said that the Court of Appeal had been wrong. They said it is not the promise that amounts to consideration but the performance of the promise. Therefore, Chandler was wrong and the buyers in this case could reclaim the full amount. Law changed by the Law Reform (Frustrated Contracts) Act 1943Restitutionary Remedy. Section 1 (2) Sums paid in advance are recoverable. Sums owed at the time of frustration but havent been paid are not paid. If there is partial performance you can still recover the amount paid in advance. If the party has incurred expenses in the performance of the contract, he is allowed to retain the advanced expenses but not in excess of this. However, this only applied if the payment is made or due before the full performance of the contract. Gamerco v ICM o Claimants paid money in advance in respect of a concert to be held in Spain. o Just before it, the authorities banned the concert due to safety concerns.

o Claimants had incurred 250,000 in expenses and the defendants had incurred 50,000. o Court held that the full advanced payment should be returned. Section 1 (3)-Discretionary power of the Courts. o Deals with part performance before the contract came to an end. o Can apply even if payment was to be made at end. o The party who conferred the benefit can receive a just sum not exceeding the value of the benefit. o BP Exploration v Bunker Hunt Justice Goff-The impact of the frustrating event must be taken into account when determining the apparent benefit that has been conferred. o Regarded by the Courts that in certain cases the cap will the be the contract price. Section 2 (3) o Parties can contract out of the provisions of this act. o It is not counted as an exclusion clause for the purpose of UCTA. Section 2 (5) o Some contracts are excluded from this act. For these contracts, the decision in Fibrosa applies.

Introduction What is the effect of withdrawing from a contract for breach? o Rescission/Termination-the injured party merely terminates the contract as to performance to the future (different from rescission for misrepresentation). Does not worry about what has taken place in the past. You may also be able to sue for damages in conjunction with this for losses that have been suffered-contract is still there and its provisions are too. (Photo Productions). o May seem the same as rescission for misrepresentation-goods returned and price recovered. o Termination for breach is not automatic. It is a right of the innocent party which he may choose to exercise. o Every breach entitles the innocent party to a remedy of damages but not always allowing them to rescind the contract. What breaches give rise to the termination of a contract? o Two methods: Nature of the term broken. Nature of the consequences of the breach. Nature of the Term Broken Sale of Goods Act-separates terms into conditions and warranties. Condition-allowed to terminate and claim damages. Warranty-only allowed to claim damages. In this method, the nature of the consequence does not matter. o Arcos v Ronasen Buyers bought products. Found that 85% of the cargo was too thick. Buyers rejected entire cargo even though it was clear that the thickness had no impact on the purpose that they were being used for. They merely rejected because they found they could buy the timber much cheaper. House of Lords held that they were entitled to do so. o Section 15A of Sale of Goods Act (companies cannot contract out of this)-buyer cannot reject if the breach is so slight that it would be unreasonable to allow him to do so. Only applies when buyer does not deal as consumer. Also only applies to breaches of conditions in Section 13-15 of the Sale of Goods Act. The burden is on the seller to show that it would be unreasonable to allow rejection. When is a Term a Condition? 1. When a statute says that it is. 2. When the courts have regularly treated it that way in a particular contract

Discharge by Breach

The Mihalis Angelos-found the date was a condition as it had been treated that way in cases like this for years. 3. If the parties expressly agree Schuler v Wickman o Stated to be condition of contract that a Wickman rep would visit the manufacturing plant at least once a week. He failed to do this and Schuler terminated contract. o Held they were not entitled to do this as condition did not mean a term that entitled the other party to withdraw in this case. This is because Schuler could terminate due to one technical breach through nobodys fault. o Dissent-this clause is the only one in the contract that was called a condition. 4. If there is none of the above: 1st Method Hong Kong Fir Shipping Company o Shipping company let to charterers for two years. Clause 3 of charter party said that owners would maintain the ship in a sufficient state beforehand. o Due to the age of the ship, it needed to be maintained by experienced staff. However, when she sailed, the chief engineer was inefficient and so she ended up out of action for many weeks. Contractors terminated the contract, thinking it would not be seaworthy in time but it was. Owners then sued. o Held that the charterers were not entitled to terminate the contract. Treating it as either a condition or a warranty seemed unfair to the parties as the delays were not serious enough to frustrate the contract. o Lord Justice Diplock (House of Lords) held that the term was neither a condition nor a warranty (innominate term-below)would the parties have thought at the time they made the contract that every breach of that term would deprive the innocent party of substantially the whole benefit which it was intended that he should obtain-if yes, then it is a condition. o This comes into play at the moment of breach which leads to uncertainty as it means that the parties did not understand the contract until this time. o However, this is incorrect due to the case below. Bunge v Tradax o Seller sold soya bean meal to supplies. Shipment from gulf of Mexico and nominated by sellers. Buyers to give at least 15 days notice of expected arrival of ship. Goods to be shipped not later than end of June.

o 17th June-buyers gave notice that loading should commence on 23rd. Sellers said the buyers had not given the 15 days notice and that the contract was terminated. Goods sold elsewhere and claimed loss in damages. o Lord Justice Megaw (Court of Appeal) held that the duty to give 15 days notice was a condition. This is because time of delivery is of the essence. o If applying the test in Hong Kong Fir they would not be able to claim that the term was a condition and virtually no term would be able to pass that test due to the high requirements. o House of Lords upheld this and so Diplocks approach is not applicable to every type of contractual undertaking. 2nd Method (Innominate Terms-the consequences of the breach) Aerial Advertising v Batchelors Peas o Contract to advertise. One term said that the company should be informed daily where they proposed to fly. o One day they forgot to notify the company and the plane ended up disturbing a silence for the end of WWI which ruined the sales for Batchelors. And so they immediately terminated the contract and sued for damages. o Held that Batchelors were entitled to terminate the contract. o The Court was applying the principle of the consequences of the breach. Not notifying of where they were flying would not always give rise to termination but in this specific case the consequences were so serious. English laws primary approach to these issues nowadays is looking at the consequences of the breach as opposed to the condition approach. Cehave v Bremer o Buyers bought pulp. On arrival, over a third was damaged and the buyers refused it. The market had fallen by time of arrival. The contract had an express term saying the goods were to be shipped in good condition. o Upon rejection, the cargo was sold by auction to X. X then resold it to originally buyers. Equally, the original buyers by spending an extra 20,000, were able to use the goods for their original purpose. o Court of Appeal held that the express term was not a condition but an innominate term and was not sufficient to terminate the contract, though they could still receive damages. This is because although implied terms are classified as warranties or conditions, there is nothing stopping them saying an express term was an innominate term. Reardon Smith Line v Hansen o Hansen ordered a tanker and the claimants chartered it in advance before it was built to Hansen.

o Before it was completed, the tanker market had collapsed. There was nothing wrong with the tanker but the charterers saw in the agreement that the description was different as tanker had a different number and was at a different port, but was exact same boat. They sought to terminate the contract for breach. o Held that there had been no breach and that the ship did comply with its description. Repudiation (Refusal to Perform) Good Faith Repudiation o Clear that it is no excuse if someone misinterprets or is advised that something they have done is not a breach of contract. o However, a good faith mistake may have some relevance: o Mersey Steel & Iron v Benzon Buyers bought 1,000 tonnes of steel a month. After first instalment, a petition was presented by creditors to wind up seller company. Buyers wrongly advised that they could not pay for the goods received without sanction of court. Sellers said they regarded this as a breach and refused further delivery. Liquidators of sellers brought action for the price of the steel and buyers counter-claimed for damaged for loss suffered by non-delivery of later instalments. Held that the delay in payment for 1st instalment did not justify sellers terminating as there was no proof that this was a repudiation of a contract. They were simply saying that the law would not allow them to pay at that time. o Woodar v Wimpey Woodar contracted to sell land to Wimpey. Contract provided that Wimpey could rescind, if prior to completion, there had been a commenced compulsory purchase of any part of property. The minister had already compulsory purchased some of it before the contract was made. Land prices then fell and Wimpey wanted to end contract. They tried to rescind due to the minister having commenced compulsory purchase proceedings and said they would not pay in the future. Court held that the clause only applied to compulsory purchase proceedings after the contract was in force. However, Wimpeys good faith was not challenged and therefore Woodars claim for damages failed. House of Lords held that Wimpey had not repudiated contract as they were simply relying on the contract (albeit in an incorrect manner). Express provision for termination

Clear provisions will be given effect e.g Union Eagle v Golden Achievement-contract for sale of flat with time of payment being of the essence. Purchasers were 10 minutes later with money and held they were entitled to terminate the contract as well as keeping the deposit received. Rice v Great Yarmouth o Claimant contracted to maintain parks. Clause said if the contractor committed a breach of any obligation the council could terminate his employment in notice by writing. o Council issued a number or warnings, saying he had committed breaches. o A few months later they terminated contract. o Court of Appeal held that the seriousness of the breaches was not sufficient to justify termination under the general law. Termination was not justifiable under their clause either as it did not make sense to read it as having a wider effect than the general law (criticism about this decision).

The process of termination Must unequivocally show intention to terminate the contract. Things such as expressly telling someone or non-performance can count. A buyer who does not pay when seller fails to deliver on time may not be found to be showing his intention to terminate the contract. Tower Hamlets LBC o Employee declines to perform full duties of employment and employer withheld pay. o Court held that this did not show that the employer had brought the contract to an end. Non-breaching party waives a right to terminate if he decides to proceed with the contract-Bentsen v Taylor. The Brimnes o Where the owners had right to terminate the contract due to not receiving payment on time. The court held the view that acceptance of late payment would have constituted an election to affirm the contract. There is no middle ground between termination and affirmation but the non-breaching party does not affirm by taking time to consider his position or calling on the party in breach to perform-Yukong Line Anticipatory Repudiation This applies where there is a refusal to perform contract in early stages or a renunciation before any performance is due. Frost v Knight o Frost engaged to Knight which was regarded as a contract. However, agreement was that they would not marry until Knights father died. Knight withdrew from agreement and Frost sued for breach of contract. o Court held that she was entitled to sue for damages regardless of the fact that the father had not yet died.

o Damages will be reduced if you accept repudiation and do not mitigate your own loss. The Mihalis Angelos o Ship chartered to carry goods. Charterers unable to get cargo due to war and so cancelled contract saying it had been frustrated. o Court held the contract was not frustrated however, charterers won because they found out that the owners had given an expected ready-to-load date which was not met and held to be a condition and so they were entitled to terminate the contract for breach. o This shows you can give a bad reason to terminate the contract and rely on a good reason of which you were unaware at the time. o Even if owner had not been in breach of condition, the Court said that any claim by him for substantial damages would have failed as the contract contained a clause allowing the charterers to cancel to contract if the vessel was late and it was clear that this would have been the case. Golden Strait Corporation o Event occurs between alleged breach and trial which would have relieved the contract breaker of liability. However, at the time of termination, this was unlikely. o In assessing the damages, the majority of the Court held that they should use the benefit of hindsight and look at what actually happened. Minority held that the damages should be assessed by looking at the time of the breach itself. White & Carter v McGregor o Claimants contracted to put up adverts for defendants on council litter bins. o Defendants repudiated agreement before performance had begun. Claimants refused to accept it and went on to do the work. (Usually be more difficult because they need the co-operation of the other party to carry out the agreement-Hounslow LBC-not entitled to continue building because it was not his property). o House of Lords held that the claimants were entitled to claim the agreed sum from the contract for doing the work. This was more a claim for a debt than for damages. o Said the case would not apply if the non-breaching party had no legitimate interest in completing the work-some confusion over this. o Said that this approach is wasteful with people doing work that others dont want. The Alaskan Trader o Law requires one to draw a line between what was merely unreasonable and wholly unreasonable-does not help us very much. The Dynamic

1. The burden is on the contract-breaker to show the innocent party has no legitimate interest in performing the contract rather than claiming damages. 2. The burden is not discharged by merely sowing the benefit to the other party is small in comparison to the loss of the contract breaker. 3. The exception to this applies only in extreme cases where damaged would be adequate and an election to keep the contract alive in unreasonable. If someone is required to mitigate their loss and suffers no loss they are not entitled to damages.

Incomplete Performance A contract to do work for B. A does not complete it or does some of it badly. B can recover damages for any damage he suffers as a result. However, if the price is payable after the work has been done, can a person refuse to pay? Cutter v Powell o Cutter was engaged as the mate of a ship for a voyage. He died during the voyage after about 70% competition. He was to be paid in one lump sum upon completion. o His widow sued for a proportionate amount. o Court rejected this, holding that the contract was an entire contract to be paid for the whole voyage and allowing damages would be inconsistent. o However, there is now legislation saying that payment of wages accrues every single day and so the Courts would decide it differently today. Equally, they could probably claim under the Frustrated Contracts Act. Sumpter v Hedges o Builder built part of house and went bankrupt (not a frustrating event) and his claim for the value of the work completed failed. o The contract had been for a certain amount upon completion of the house. o Here the Frustrated Contracts does not apply and neither can it be said that the owner had to pay for the partially built house as he did not voluntarily accept this. Hoenig v Isaacs o Defendant contracted for decoration of flat including installation of built in furniture. Some of the amount had already been paid and most of the work had been done when the claimant said he could not build any more. o It would cost extra money to put some defects wrong and the defendant refused to pay the balance owed. o Court of Appeal held that there had been substantial performance and he was entitled to the balance minus the amount needed to restore the defects.

Bolton v Mahadeva o Mahadeva contracted for the claimant to install central heating where he would receive the full amount at the end. o System didnt work properly and evidence showed it would cost to repair. Claimant refused to do the necessary work. o Court rejected his claim for payment entirely, saying his case lay on the opposite side to Hoenig. o Question over whether this is fair-Mahadeva received a fully functioning heating system for much cheaper while the installer received nothing.

Introduction This does not deal with self-help remedies such as rescission for misrepresentation. Divided between: o Specific relief-enforced performance o Substitutionary relief-something in place of performance such as damages Common law is reluctant to award specific relief-damages are primary remedy for breach of contract. Damages Award of damages deals with 3 interests: o Expectation interest-different principle-putting in the situation if the contract had been performed. Deals with breach of contract. Robinson-Put them in the monetary equivalent, as far as it is possible that they would have been in if the contract had been performed. o Reliance (tort) interest-wasted expenditure and putting him back in situation before contract. Deals with fraudulent misrepresentation-tort. Price paid less value received o Restitution interest-overlaps with unjust enrichment. Only given if there is a total failure of performance which is rare. Expectation Damages Adopted as a cheap, efficient rule and is more practicable than reliance damages. However, sometimes the expectation damages may over compensate e.g. if the contract was repudiated 20 second after it was made. In terms of the duty to mitigate you loss your receive damages for the difference between the price of the thing you bought and the substitute you needed to buy when the goods were not delivered. East v Maurer o Vendor sold one hairdressing salon. He still practiced in another one. o Vendor said he was going to live abroad which was a lie and he continued to operate the other salon to the detriment of the purchaser. o Held that there was fraudulent misrepresentation. In terms of damages, the Court could not award for breach of contract (as there was no contractual term dealing with this) but he could receive money that he could have invested elsewhererecovery of profits. Expectation damages: diminution in value vs. cost of cure-SET OUT BOTH IN EXAM Radford v de Froberville

Judicial Remedies For Breach Of Contract

o Defendant bought land and agrees that he will build a wall on the boundary of the land. Defendant then sells land to someone else. o Wall never built. Absence of the wall did not reduce the rental income. Defendant argued that the damages should be nominal. Specific performance could not be awarded. o Awarded claimant damages representing the cost of building the wall himself-example of cost of cure. Jacob v Youngs & Kent o House owner withheld final balance of payment because different piping was installed than contracted for (even though it was exactly the same). o Court held that the builder could recover the full amount. Ruxley v Forsyth o Judge held that a shallower pool did not affect the value of the property and was also safe for diving. o House Lords held that the cost of rebuilding the pool was unreasonable but that he could be awarded 2,500 as part of the diminution in value taking into account his loss of amenity. o They did not give him the cost of cure as there was no guarantee he would actually rebuild the swimming pool as he was in the process of selling his house. In the majority of cases the cost of cure will be the standard form of damages.

Reliance Damages This is an alternative to the expectation measure. McRae v Commonwealth Disposals Commission o Case of the non-existent tanker on the non existent reef which was sold. o Held the contract was not void for mistake but that the commission was in breach of contract. o In terms of damages, as there had been no description of the tanker at all, it was impossible to assess the profits that would have been made. And so he based his claim on the wasted expenses he incurred going to find the boat etc. Up to the defendant to show that the claimant would have made a loss which will then be set off against the damages. CCC Films v Impact Quadrant Films o Defendants failed to carry out contract to send tapes by insured, recorded delivery. They were lost in transit. o Claimants were not able to produce evidence of likely receipts from sales of the films but neither could the defendants. o As the burden of proof was not on the defendant, they could not show that the claimant would have made a loss and so the claimant received full amount of damages. Haulage v Middleton o Haulage allowed Middleton use of garage premises terminable upon 6 months notice.

o However, they excluded Middleton from the premises without the required notice. o Middleton was able to carry on his business elsewhere and suffered no loss of trading receipts. He claimed money for labour and materials he used in altering the premises he was kicked out of. Under terms of license, Middleton could not remove fixtures from the building and so the improvements would have been Haulages when they received the premises back again. o Court of Appeal dismissed claim for wasted expenses as even if Haulage had acted lawfully, the expenditure would still have been wasted as it would have benefitted Haulage. Anglia v Reed o Anglia planned to make film and incurred expenses before signing up Reed in lead role. Within 2 weeks, Reed repudiated and film was cancelled. o Anglia claimed damages for money spent before Reed signed. Reed admitted liability but only for expenses suffered after he signed the contract. o Anglia could not claim on expectation damages as they had no way of proving how much would have been made. o The claim for damages succeeded as the expenditure was wasted because of his breach of contract. Cullinane v Rema o Buyer wanted to buy machine which processed 6 tonnes of clay an hour. This turned out to not be true and the machine was unprofitable. o Said that a claimant can either claim reliance damages for wasted expenditure on the machine but alternatively he can claim on the basis of the profit he has lost due to the machine.

The Restitution Interest 1. Can recover the full price paid in advance for goods that were never delivered. This is not a claim for damages and therefore proof of loss is irrelevant-seller cannot keep some of the price to reflect a loss that would have been made- Bush v Canfield. 2. Quantum meruit gains-if work is done for a person who then repudiates. The work has conferred a benefit on the person and so the other can recover the value of the work. Again this is not a claim for damages. Teacher v Calder-timber merchant in breach of a contract took some of his capital out of a business and used it to buy distillery which made large profits. Claimant claimed a share of the profits but it failed because the money withdrawn was not his in any sense and there was no evidence of any lost business as a result. It is claimants loss not the defendants gain. However, if there was a fiduciary

relationship between the two, then the outcome would have been different. Waiver of Tort o You could opt out of taking a tort claim and turn to restitution instead. If he stole a Rolex which was worth 1,000 and sold it to someone who paid more than it was worth. Here I could drop the tort claim of conversion and instead sue for the amount made by the robber. Wrotham Park v Parkside o Defendant in breach of a restricted covenant, built more houses on land then he should have. o Firstly, the claimants sought injunction for the houses to be destroyed but were refused. o In lieu of this, the court awarded damages of 5% of the profit made from them. Upheld in the case below. A-G v Blake o Blake was spy for Soviet, sentenced to prison, managed to escape. o In breach of contract he wrote his memoirs which were published in UK and an action was brought to prevent him receiving any royalties. o Court of Appeal took the economic efficiency route and said that Blake should be allowed to benefit and keep the profit. o House of Lords took the moral route and held that in some cases, account of profits can be used as a remedy. However, this should apply only in exceptional circumstances. It is more of a punishment than normal damages. o Lord Nicholls-general rule is asking whether the claimant had a legitimate interest in preventing the defendants profit. Held that this specific case proved this. Esso v Niad o Esso undertook to match price of supermarkets in the sale of petrol. Made an undertaking to give their franchisees money to ensure they could accommodate this promise. o Niad sold petrol for money higher than the supermarket price whilst keeping the money Esso gave him and Esso sued for breach of contract claiming the profit he made. o Court held that this was a case where Blake is applied as Esso had a legitimate interest in preventing Niad making a profit. Experience Hendrix LLC v PPX o Breaches by PPX of settlement agreement on right to exploit Hendrix recordings. Claimants could not show they had suffered any loss as a result. o Deliberate breach of commercial contract is not enough to give claimant a legitimate reason for claiming the profits. o The interests of the Claimants deterring the breaches was not as strong as that in Blake or Esso.

o Held that an account of profits was not justifiable but that it was justifiable to award a fair royalty payment on the sale of the recordings under the principle in Wrotham Park. o Wrotham Park damages are not confined when they are given in lieu of an injunction. Considered that Wrotham Park damages are as a compensatory remedy and are very similar to account of profits-World Wide Fund v WWF.

Intangible Losses No damages are recoverable for injury to feelings and exemplary damages are never available for breach of contract-Addis v Gramophone-claim for humiliating, wrongful dismissal. Damages for distress etc can be recoverable when a significant object of the contract was to provide enjoyment, peace of mind etc. o Jarvis v Swans Tours-dealt with a holiday where the ambiance was depressing and not as advertised. Court of Appeal awarded damages as the thing was so disappointing. o Farley v Skinner-purchaser of house asked surveyor to consider aircraft noise near Gatwick. Contrary, to report, was found that there was significant noise from aircraft waiting to land. Noise was not great to have any measurable value on house and claimant was not hypersensitive but found it annoying. House of Lords upheld damages due to the fact that it was an important element of the contract which justified the award of more than token damages. Remoteness of Damage Hadley v Baxendale o Baxendale contracted to carry Hadleys crankshaft to be repaired. It was late upon return and the mill was closed down for a number of days. o Hadley claimed for the loss of profits during those days. o Claim was rejected. o 2 Limbed Test: Losses that flow from naturally from the breach. Losses that are claimed to have arisen due to the breach. o The damages should be such as may fairly and reasonably be considered arising naturally-according to the usual course of things from such a breach of contract or such as may reasonable be supposed to have been in the contemplation of both parties at the time they made the contract. o Knowledge must be possessed at the time the contract was made.

o Defendants did not know that the mill would be shut until the crankshaft was repaired and returned. Victoria Laundry v Newman o Claimants bought new boiler which was delivered 5 months late but still decided to accept it. They then claimed damages for both loss of new business they could have taken on as well as for loss of lucrative dyeing contracts. o Court of Appeal allowed damages for loss of business as no business would buy this massive product except to put it into use in its business. Court said the assumption by the defendants that the boiler was a spare was silly. o They rejected the claim for the dyeing contracts, as the defendants were unaware of those potential contracts. o Court said the claimants are not precluded from recovering a general sum for loss of business from dyeing contracts to be reasonable expected. o Damage simply needs to be on the cards as a consequence of the breach. The Heron II o Case dealt with whether carrier of goods can be liable for loses attributable to changes in the market due to delay. o House of Lords held that they can be liable. o Criticism made of on the card approach above as it was too remote a chance. The loss must be more likely than the reasonably foreseeability of negligence law. Parsons v Uttley Ingham o Defendants supplied equipment to hold pig nuts but were negligent and nuts went mouldy and the pigs got ill and over 250 died. o Not likely pigs would die but that they would become ill and Court of Appeal upheld decisions that the defendants were liable to the farmer. Not consistent with Victoria Laundry. o Lord Denning-The Heron was never intended to apply where breach of contract caused physical damage. In this cases the same approach on foreseeability as in tort applies. o Other judges felt the general nature of the damage must be in the mind of the parties and not the specific facts. Transfield Shipping v Mercator Shipping (The Achilleas) (Main case) o Owners chartered a ship for 5-7 months. The last date for redelivery was 2nd May. On 20th April, charterers gave notice of redelivery and so owners rechartered ship to another company. o However, ship was delayed and not redelivered for over 10 days. New charterers had chance to end contract but instead negotiated lower charter rate. o Owners sued the charterers for damages for the difference between the original new charter rate and the re-negotiated rate throughout the second charter. Charterers said it should be based on market value during delay.

o House of Lords held that the charterers were only liable for the market rate due to the fact that in contract, unlike in tort, responsibility is voluntarily undertaken. Lord Hoffman- The question is whether one can conclude that the parties when making the contract, regarded a consequence as one which the contract breaker took responsibility for. In this case, the contemplation of the parties when thinking of delay would be the loss of money at the market rate. Mitigation 1. Loss which would have been recoverable but which has been avoided is not recoverable-Erie Country Natural Gas v Carroll (claimant sold natural gas rights but detained right to enough gas to run their business, defendant broke contract, claimants purchased further leases at cost and eventually sold them at a profit-there action had mitigated their loss). 2. Loss which could have been avoided by taking reasonable steps is not recoverable-standard required of the claimant is not high. Findlay v Kwik Hoo Tong (shipment under falsely dated bill of lading, if Findlay had known they would have rejected cargo, Findlay resold cargo to new company who found out about the bill of lading and refused it, Findlay sued Kwik Hoo, decided that Findlay should not have been required to mitigate their loss). Contributory Negligence The Law Reform (Contributory Negligence) Act 1945 does not apply to claims for breach of contract unless the duty is to take reasonable care and on the facts there would be a tortuous duty even without the contractual provision. It is an all or nothing approach when it comes to causation. Liquidated (Agreed) Damages Desirable that parties agree at outset, the amount paid if the contract is broken to avoid costs of enquiring into damages. Law draws distinction between liquidated damages and penalties. Dunlop Pneumatic Tyre v New Garage (above) o Evidence that if tyres got onto the market at cost price, it would be damaging to the company and would weaken their reputation. o House of Lords allowed the specification of 5 as damages for every breach. o Validated liquidated damages clause is binding if it was genuine attempt to predict the loss and therefore the specified sum is payable regardless of what the actual damage was. o Does not matter if it is impossible to predict the damage. The penalty is only enforceable up to the damage actually suffered. Murray v Leisureplay

o Director was employed under contract which entitled him to one years notice and that if this was breached he was entitled to one years pay. o Held that this clause was not a penalty as it was not a genuine attempt to estimate the loss suffered, as it did not take into account potential re-employment prospects. o Seems to be a move towards an enquiry into whether the clause is unreasonable or unconscionable. o Cases emphasises that the burden is on the person attacking to clause.

Equitable Remedies
Specific Performance Courts can now award damages in lieu of specific performance. Specific performance will be awarded if damages would not be adequate. Can only be awarded in sale of specific goods but not generic goods as you can simply go out and buy the product elsewhere and receive damages-Sale of Goods Act 1979. Usually only awarded if the person cannot create a new contract with someone else for the same thing. Different to European system (Lando) where specific performance is the primary remedy for breach of contract. Failure to comply with a specific performance is contempt of court-courts might be reluctant to use coercive power of state to try and get people to keep their contractual obligations. Sky Petroleum o Seller contracted to fill buyers requirements of petrol for 10 years. During shortage of petrol due to war, the sellers could get higher prices elsewhere and said they were terminating agreement. o Buyers were awarded an injunction to stop them breaking the contract instead of specific performance. Some instances when the court will never award specific performance-court will never order an employee to perform contract of service/promises made by deed. It often depends on how complicated and difficult it would be to perform the contract. However, we see specific performance being the norm for cases such as the sale of land. English law regards specific performance as a secondary or exceptional remedy unlike civil law systems such as Germany. General Discretionary Considerations: o Claimants misconduct: Specific performance will not be ordered if the claimant is entitled to rescind the contract for misrepresentation etcRedgrave v Hurd.

However, misconduct less than this may lead to refusal also-Quadrant Visial Communication-(underhanded conduct in issuing vouchers which damaged the other business before a sale. o Hardship to defendant: Not enough to show that the defendant has made a bad bargain and lost money. Tamplin v James Man bids at auction and wins but finds out he hasnt bought what he thought he was getting as he didnt read the auction particulars. Vendor knew nothing about this mistaken belief. Court decreed specific performance. Hope v Walter Defendant contracted to buy a house which had sitting tenant. Before completion he found out it was being used as brothel. Vendors sought specific performance. Held that it would be wrong to decree specific performance as it would cause hardship to the defendant o Mutuality Mutuality requirement is to be judged at the time of the claim and the basic question is whether it may be granted without unfairness to the defendant. There is a presumption that it will be refused if at the time of the claim the party against whom it is sought would have no security for the performance by the other beyond a claim for damages. o Constant Supervision Ryan v Mutual Tontine Lease contained term requiring landlord to provide resident landlord in block of flats. Specific performance was not granted as it would need constant supervision. Cooperative Insurance Society v Argyll Stores Safeway took a lease of supermarket in shopping centre. A clause required them to keep the premises open for the normal hours of business. Safeway faced fierce competition and decided to close 27 loss-making stores and one of them was the one in question. They told the landlords that the shop would close and the fittings were stripped out. Safeway was willing to pay the whole of the rent for the balance of the lease. However, landlords unhappy as it would cause loss of customers to shopping centre overall. House of Lords refused to offer specific performance. Held that the court is not actually

required to supervise specific performance as the claimant would bring any breaches in front of it but that there could be long series of returns to the courts as a result. Also for a breach it leads to contempt of court which carries a prison sentence which is a very serious issue which the courts do not wish to create.

1. a-Uncertainty-May v Butcher-Agreements to agree. Only allowed to say at a price to be agreed as long as you provide a fallback such as it being decided by 3rd party. Saying not less than 5,000 is not a sufficient fallback. Foaly-inconsistent with may v butcher. b-Mistake-obvious to landlord and cannot enforce it. 2. Part payment-high trees over foakes v beer. Law in uncertainty. Privity of contract-agreement made with David not olwen. High trees estoppel could not be covered by Contract Rights of Third Parties Act as it is not a contract. 4-Not really mistake. Misrepresentation and Breach. Oscar chess-deciding on whether a statement is a representation or a term of the contract is down to the relative knowledge of the parties at the time-she was not educated in card and so was a mere representation. Misrep-damages put back in position he was. Breach of condition-expectation damages. 6.-Conditions/Warranties. a-Arcos v Section 15A (only applies to implied descriptions about quality not delivery date). 7-A-Frustration-Law Reform Frustrated Contracts Act-fact that Cozy can retain all or part f the advanced payment and also the fact that a benefit has be conferred upon the other party. B-breach of contract-advanced payment is standing. No damages as he has been given advantage. 8-A-A-G V Blake. B-Remoteness of damage-achilles/

You might also like